NURS 223: Exam 3

Réussis tes devoirs et examens dès maintenant avec Quizwiz!

A patient questions the nurse about herbal treatments for arthritic pain. What is the nurse's best response? 1 "Glucosamine has shown promise. Consult your healthcare provider." 2 "Ginkgo biloba has proven to be very useful as an anti-inflammatory agent." 3 "There are no safe herbal treatments for pain; take your prescription medications." 4 "High doses of vitamins have been used for many years to help maintain joint health."

1) "Glucosamine has shown promise. Consult your healthcare provider." Glucosamine has been used to help osteoarthritic pain; however, herbal medicines should always be used in consultation with the healthcare provider.

A patient newly diagnosed with type 2 diabetes mellitus has been ordered insulin glargine [Lantus]. Which information is essential for the nurse to teach this patient? 1 "This medication has a duration of action of 24 hours." 2 "This medication should be mixed with the regular insulin each morning." 3 "This medication is very expensive, but you will be receiving it only a short time." 4 "This medication is very short-acting. You must be sure you eat after injecting it."

1) "This medication has a duration of action of 24 hours." Insulin glargine [Lantus] has a duration of action of 24 hours with no peaks, mimicking the natural, basal insulin secretion of the pancreas. This medication cannot be mixed with other insulins and is not a short-acting insulin. The patient may need to receive this medication for a long time.

The nurse should include which statement when teaching a patient about insulin glargine [Lantus]? 1 "You cannot mix this insulin with any other insulin in the same syringe." 2 "You should inject this insulin just before meals because it is very fast acting." 3 "You can mix this insulin with neutral protamine Hagedorn (NPH) insulin to enhance its effects." 4 "The duration of action for this insulin is approximately 8 to 10 hours, so you will need to take it twice a day."

1) "You cannot mix this insulin with any other insulin in the same syringe." Insulin glargine [Lantus] is a long-acting insulin with a duration of action up to 24 hours. It should not be mixed with any other insulins. The insulin is not fast acting.

When administering a medication known to be metabolized by the liver, the nurse should closely monitor for adverse drug reactions in which patient? 1 A 3-month-old infant 2 A 12-month-old infant 3 An 18-month-old toddler 4 A 13-year-old adolescent

1) A 3-month-old infant Neonates are especially sensitive to drugs that are eliminated primarily by hepatic metabolism. When these drugs are used, dosages must be reduced. The capacity of the liver to metabolize many drugs increases rapidly about 1 month after birth and approaches adult levels a few months later. The liver has matured completely by 1 year of age.

How do H2 receptor antagonists reduce the secretion of acids? 1 By blocking the H2 receptors 2 By stimulating gastrin receptors 3 By blocking acetylcholine receptors 4 By stimulating acetylcholine receptors

1) By blocking the H2 receptors H2 receptor antagonists block H2 receptors and stop the secretion of acids by parietal cells. H2 receptor antagonists do not block acetylcholine receptors. They make parietal cells unresponsive to the stimulation of acetylcholine and gastrin receptors. H2 receptor antagonists do not directly affect gastrin and acetylcholine receptors.

The patient is having an overnight dexamethasone suppression test performed to diagnose Cushing's syndrome. A nurse administers dexamethasone [Decadron] 1 mg, at 11:00 PM. During shift change report, the nurse should instruct the oncoming nurse to draw blood for which test at 8:00 AM? 1 Cortisol 2 Sodium 3 Glucose 4 Potassium

1) Cortisol The overnight dexamethasone suppression test is used to diagnose Cushing's syndrome. Normally, dexamethasone acts to suppress the release of adrenocorticotropic hormone (ACTH), thereby suppressing the release of cortisol. In a patient with Cushing's disease, no cortisol suppression occurs. Potassium, glucose, and sodium are not used as measures of adrenal function.

While assessing the renal function in an older adult patient, a nurse learns that the patient has reduced renal blood flow. What is the impact of this age-related change on drug action? 1 Decreased excretion 2 Decreased metabolism 3 Increased protein binding 4 Increased first-pass metabolism

1) Decreased excretion The impact of reduced blood flow to the kidney due to increasing age results in decreased drug excretion. Therefore, the drug may remain in the blood for a relatively long time. As a result, the dosage would need to be adjusted. Reduced renal blood flow in older adult patients does not decrease metabolism of the drug, because it usually happens in the liver. The renal system has no role in protein binding or the first-pass metabolism of the drug.

The nurse working with elderly patients is concerned about the number of medications each patient is taking. Which should the nurse assess as the highest priority for the patients related to polypharmacy? 1 Drug interactions 2 Cost of medications 3 Schedule of medications 4 Nonadherence to drug regimen

1) Drug interactions The highest priority for patients with multiple medications (polypharmacy) is the assessment for drug interactions. The more medications a patient takes, the higher is the risk of drug interactions.

A patient tells the nurse that she is thinking about becoming pregnant. The nurse teaches the patient that which vitamin should be her priority for supplementation before planning a pregnancy? 1 Folic acid 2 Vitamin D 3 Riboflavin 4 Vitamin B12

1) Folic acid Folic acid deficiency during early pregnancy can cause neural tube defects (spina bifida). All women with the potential for becoming pregnant should consume folic acid every day. Vitamin B12, riboflavin, and vitamin D are not considered as important as folic acid to supplement before a woman becomes pregnant.

A patient who is obese is newly diagnosed with type 2 diabetes and is instructed to make dietary changes and perform aerobic exercises daily. The patient insists on insulin therapy for prompt management of diabetes. However, the patient is prescribed oral medications for lowering glucose levels. Which is the most likely reason for delaying insulin therapy? 1 Insulin therapy is initiated when other methods have failed. 2 Insulin therapy is ineffective without initial oral drug therapy. 3 Insulin therapy is not generally prescribed for obese patients. 4 Insulin therapy may cause congestive heart failure to develop.

1) Insulin therapy is initiated when other methods have failed. For patients with type 2 diabetes mellitus, insulin therapy is usually reserved for situations when other treatment methods are unsuccessful. It is more important to start oral drug therapy and initiate lifestyle changes as an initial treatment for diabetes mellitus management. Insulin therapy does not cause congestive heart failure in patients. It is more a concern in patients who take pioglitazone [Actos]. Insulin is prescribed for obese patients if the initial drug therapy and lifestyle changes are ineffective in lowering glucose levels. Insulin therapy is used for diabetic management and is used along with oral drug therapy.

A patient newly diagnosed with type 1 diabetes asks a nurse, "How does insulin normally work in my body?" The nurse should base his or her response on which understanding of the mechanism of insulin? 1 It promotes the passage of glucose into cells. 2 It stimulates the pancreas to reabsorb glucose. 3 It stimulates the liver to convert glycogen to glucose. 4 It promotes the synthesis of amino acids into glucose.

1) It promotes the passage of glucose into cells. The hormone insulin promotes the passage of glucose into cells where it is metabolized for energy. Insulin does not stimulate the pancreas to reabsorb glucose or synthesize amino acids into glucose. It does not stimulate the liver to convert glycogen into glucose.

A patient has a prescription for an oral glucocorticoid. Which glucocorticoid will the nurse administer? 1 Prednisolone acetate 2 Triamcinolone hexacetonide 3 Betamethasone sodium phosphate 4 Methylprednisolone sodium succinate

1) Prednisolone acetate Of the glucocorticoids listed, prednisolone acetate is the only one that can be given orally. Triamcinolone hexacetonide is a local glucocorticoid that can be given by the intra-articular or intralesional route. Betamethasone sodium phosphate and methylprednisolone sodium succinate can be given IM or IV. Additionally, betamethasone sodium phosphate can be given by the intra-articular, intralesional, and soft tissue routes.

Which intervention should the nurse choose to minimize the risk of drug toxicity when giving protein-bound drugs to neonates and infants? 1 Reduce the amount of drug given. 2 Shorten the interval between doses. 3 Administer the medication intravenously. 4 Administer the medication before feedings.

1) Reduce the amount of drug given. The albumin in neonates and infants has a lower binding capacity for medication. A lower binding capacity leaves more of the free drug available for action; therefore, a lower dose is required to prevent toxicity.

A nurse is assessing a child with an inadequate intake of vitamin D. Which condition will the nurse monitor for in this child? 1 Rickets 2 Anemia 3 Pellagra 4 Osteomalacia

1) Rickets The classic effects of deficiency are rickets (in children) and osteomalacia (in adults). Anemia can occur with vitamin B12 and folic acid deficiencies. Pellagra is a deficiency of niacin (nicotinic acid).

When preparing a teaching session for residents at an assisted living facility, what should the nurse include? 1 The importance of taking the full prescribed dose 2 The importance of taking double amounts of missed doses 3 The importance of reducing protein intake while taking prescription medications 4 The importance of using multiple pharmacies for cost-effective savings on prescription drugs

1) The importance of taking the full prescribed dose Underdosing, with resulting therapeutic failure, is much more common (90%) than overdosing among the elderly. In most cases (75%), the nonadherence is intentional because of the patient's conviction that the drug is simply not needed or because of unpleasant side effects. Using multiple pharmacies should be avoided, as should doubling missed doses. Doubling a dose could result in intentional overdosing. Reducing protein intake can result in decreased drug binding to albumin; consequently, the amount of free drug is increased, which could result in drug toxicity.

A patient's body mass index (BMI) is 35. How should the nurse interpret this finding? 1 The patient is obese. 2 The patient is overweight. 3 The patient is underweight. 4 The patient is of normal weight.

1) The patient is obese. According to the federal guidelines, a BMI of 30 or higher indicates obesity. Individuals with a BMI of 25 to 29.9 are considered overweight but not obese. A healthy BMI is 18.5 to 24.9 while underweight would be below 18.5.

A patient is scheduled to start taking insulin glargine [Lantus]. Which information should the nurse give the patient regarding this medication? 1 This medication provides blood glucose control for 24 hours. 2 The peak effect of this medication is achieved in 2 to 4 hours. 3 This medication provides mealtime coverage of blood glucose. 4 Less frequent blood glucose monitoring is required when taking this medication.

1) This medication provides blood glucose control for 24 hours. Insulin glargine is administered as a once-daily subcutaneous injection for patients with type 1 and type 2 diabetes. It is used for basal insulin coverage, not mealtime coverage. It has a prolonged duration, up to 24 hours, with no peaks. Blood glucose monitoring is still an essential component to achieve tight glycemic control.

Which information should the nurse include in the teaching plan for a patient who has been prescribed repaglinide [Prandin]? 1 "This medication will not cause hypoglycemia." 2 "You will need to be sure you eat as soon as you take this medication." 3 "You do not have to worry about side effects when taking this medication." 4 "When taking this medication, use aspirin rather than acetaminophen for pain relief."

2 "You will need to be sure you eat as soon as you take this medication." Repaglinide [Prandin] is short acting. The drug's very fast onset of action allows patients to take the drug with meals and skip a dose when they skip a meal. The effects of repaglinide [Prandin] are enhanced when taken with aspirin and other nonsteroidal anti-inflammatory drugs (NSAIDs), which may result in increased hypoglycemia. Hypoglycemia is a side effect of this medication, and there are many other possible side effects of this medication.

A nurse instructs a female patient with peptic ulcer disease who is to start a treatment regimen that includes ranitidine [Zantac] and bismuth subsalicylate [Pepto-Bismol]. Which statement by the patient indicates that the teaching has been effective? 1 "I'll include more calcium and vitamin D in my diet to prevent osteoporosis." 2 "While I'm taking these medications, my bowel movements could look black." 3 "I'm so glad that my allergies will be helped while I'm taking these medications." 4 "I have a medicine at home to take when I start having some loose diarrhea stools."

2) "While I'm taking these medications, my bowel movements could look black." Regimens for eradicating H. pylori include using two or three antibacterials with an antisecretory agent or histamine2 receptor antagonist. Bismuth acts topically to disrupt the cell wall of H. pylori. It can cause a harmless black stool discoloration. Loose stools are an adverse effect of systemic antibacterials such as amoxicillin [Amoxil]. Ranitidine [Zantac] produces selective blockade of H2 receptors, which inhibits gastric acid secretion only, not allergy symptoms. Osteoporosis is an adverse effect of omeprazole, a proton pump inhibitor (PPI).

The healthcare provider orders 150 mcg of levothyroxine [Synthroid] PO every morning. The medication available is levothyroxine [Synthroid] 75 mcg tablets. How many tablets will the nurse administer? 1) 1 2) 2 3) 4 4) 0.5

2) 2 The ordered dose is 150 mcg. The available tablets are 75 mcg; 75 multiplied by 2 equals 150. Therefore, two tablets are the correct dose.

A nurse is reviewing the medication records for vitamin K. The nurse will hold vitamin K and call the provider regarding which patient? 1 A newborn infant 2 A patient taking warfarin 3 A patient with celiac disease 4 A patient with megaloblastic anemia

2) A patient taking warfarin Vitamin K is not given to a patient taking warfarin, as this will interfere with the action of the warfarin. There is no contraindication to administering vitamin K for the other patients.

Which is the nurse's best action when finding a patient with type 1 diabetes mellitus unresponsive, cold, and clammy? 1 Start an insulin drip. 2 Administer intravenous glucose. 3 Draw blood glucose level and send to the laboratory. 4 Administer subcutaneous regular insulin immediately.

2) Administer intravenous glucose. This patient is showing signs of hypoglycemia. In the hospital setting or when the patient is unconscious, intravenous glucose is an obvious option to treat hypoglycemia.

The nurse prepares to administer a newly prescribed medication to a 22-year-old woman. The insert in the medication package states, "Pregnancy Category X." Select the nurse's best action. 1 Assume that the patient is pregnant. 2 Ask the patient, "When was your last menstrual period?" 3 Assess the patient for a history of sexually transmitted disease. 4 Ask the patient, "Have you been sexually active during the past year?"

2) Ask the patient, "When was your last menstrual period?" Category X means that the drug will be harmful to the fetus if the patient is pregnant. The patient may not know whether she is pregnant; therefore, asking her when her last menstrual period occurred gives the nurse a better indication of whether the patient might be pregnant.

A nurse instructs a patient taking long-term glucocorticoid therapy to have eye examinations every 6 months. What is the nurse trying to prevent? 1 Infection 2 Cataracts 3 Myopathy 4 Cushing's syndrome

2) Cataracts Cataracts are a common complication of long-term glucocorticoid therapy. To facilitate early detection, patients should undergo an eye examination every 6 months. Myopathy is muscle injury manifesting as weakness usually in the proximal muscles of the arms and legs. Long-term glucocorticoid therapy can induce a cushingoid syndrome with symptoms identical to those of naturally occurring Cushing's syndrome. Although long-term glucocorticoid therapy can cause infection, that is not the reason to recommend eye examinations every 6 months.

Which factor in a patient's history is most likely related to the development of thiamine (vitamin B1) deficiency? 1 Exposure to asbestos 2 Chronic alcohol abuse 3 Gastric resection surgery 4 Having a heart transplant

2) Chronic alcohol abuse Thiamine deficiency is common in individuals who abuse alcohol. Thiamine requirements are related to caloric intake, and principal dietary sources are enriched, fortified, or whole-grain products, such as breads and cereals. Exposure to asbestos, heart transplantation, and gastric resection surgery are situations unrelated to thiamine deficiency.

The nurse is planning care for a patient with signs of acute adrenal insufficiency. What is the priority patient problem? 1 Pain and discomfort 2 Deficient fluid volume 3 Excessive caloric intake 4 Shortness of breath upon exertion

2) Deficient fluid volume Acute adrenal insufficiency (adrenal crisis) is characterized by hypotension, dehydration, weakness, lethargy, and gastrointestinal symptoms of nausea and vomiting. Rapid replacement of fluid, salt, and glucocorticoids is essential to prevent shock and death. Comfort, nutrition, and activity are important to address once fluid balance has been restored.

The safety or potential harm of drug therapy during pregnancy relates to which factor? 1 Fetal sex 2 Drug properties 3 Diet of the mother 4 Maternal blood type

2) Drug properties Drug properties have a direct correlation to the safety or potential harm of drug therapy during pregnancy, and nurses working in prenatal settings need to be aware of information related to drug properties.

A patient going on a vacation cruise is prescribed a scopolamine transdermal patch for motion sickness. The nurse teaches the patient to recognize which side effect? 1 Irritability 2 Dry mouth 3 Urinary frequency 4 Increased heart rate

2) Dry mouth Scopolamine blocks the binding of acetylcholine with cholinergic receptors in the inner ear, an imbalance that is a common cause of motion sickness. The most common side effects are dry mouth, blurred vision, and drowsiness. Urinary retention occurs less frequently. An increased heart rate and irritability are not associated effects.

A pediatric patient has gigantism caused by excess growth hormone (GH). Which finding would indicate to the nurse that the patient has developed an additional complication related to this condition? 1 Atrophy of sweat glands 2 Enlarged heart on chest x-ray 3 Blood glucose below 70 mg/dL 4 Elevation of liver function test results

2) Enlarged heart on chest x-ray Gigantism caused by excess GH can cause children not only to grow very tall but also to develop complications such as headache, profuse sweating, cardiomegaly (enlarged heart), and diabetes. Because of its effect on carbohydrate metabolism, excess GH may cause an elevated blood glucose level, not hypoglycemia. It does not damage the liver; therefore, liver function tests are not affected. It also does not cause the sweat glands to atrophy.

Which is an effect of octreotide [Sandostatin]? 1 It stimulates the body's metabolism. 2 It antagonizes the effect of growth hormone. 3 It stimulates the protein synthesis in the muscles. 4 It regulates the fat metabolism in adipose tissues.

2) It antagonizes the effect of growth hormone. Octreotide [Sandostatin] is a drug that antagonizes the effects of the natural growth hormone.

A patient who has gastroesophageal reflux disease (GERD) is taking magnesium hydroxide (milk of magnesia). Which outcome should a nurse expect if the medication is achieving the desired therapeutic effect? 1 Reduced duodenal pH 2 Neutralized gastric acid 3 Reduced stomach motility 4 Increased barrier to pepsin

2) Neutralized gastric acid Antacids work by neutralizing, absorbing, or buffering gastric acid, which raises the gastric pH above 5. For patients with GERD, antacids can produce symptomatic relief. An increased barrier to pepsin is an effect of sucralfate [Carafate]. Reduced stomach motility is not an effect of milk of magnesia. A reduced duodenal pH would result in a more acidic environment.

Which statement is found in the Dietary Supplement Health and Education Act (DSHEA) of 1994? 1 No proof of efficacy is required for legend drugs. 2 No proof of efficacy is required for dietary supplements. 3 No proof of efficacy is required for over-the-counter drugs. 4 Dietary supplements have to be supplied upon prescription.

2) No proof of efficacy is required for dietary supplements. According to the Dietary Supplement Health and Education Act (DSHEA) of 1994, no proof of efficacy is required for dietary supplements because they are considered to be safe. In contrast to this, proof of efficacy is required for legend drugs and over-the-counter drugs. These drugs have a narrow therapeutic index when compared to dietary supplements. Prescription is not required for selling and supplying dietary requirements to patients because they have a greater margin of safety.

A patient with hepatic encephalopathy receives lactulose. The nurse expects which therapeutic outcome if the medication is having the desired effect? 1 Less ascitic fluid 2 Reduced ammonia level 3 Release of glycogen stores 4 Normal serum sodium level

2) Reduced ammonia level Some practitioners use lactulose to reduce blood ammonia levels by forcing ammonia from the blood into the colon. Lactulose is useful for treating patients with hepatic encephalopathy. It does not result in less ascitic fluid, a normal serum sodium level, or release of glycogen stores.

When assessing for potential adverse effects in a patient taking glucocorticoid and digoxin [Lanoxin], what is a priority assessment? 1 Serum calcium levels for hypercalcemia 2 Serum potassium levels for hypokalemia 3 Serum sodium levels for potential hyponatremia 4 Intake and output for potential fluid volume deficit

2) Serum potassium levels for hypokalemia Glucocorticoids can increase urinary loss of potassium and can thereby induce hypokalemia. Consequently, glucocorticoids must be used with caution when combined with digoxin [Lanoxin] (because hypokalemia increases the risk of digoxin-induced dysrhythmias). Calcium levels are not a concern when glucocorticoid and digoxin [Lanoxin] are given concurrently. Hypernatremia and fluid retention can occur, not hyponatremia or fluid volume deficit.

A pregnant woman admits taking a friend's left-over antibiotics for a sore throat. The nurse will need to verify that the antibiotic is void of what kind of effects? 1 Allergic effects 2 Teratogenic effects 3 Carcinogenic effects 4 Hallucinogenic effects

2) Teratogenic effects Teratogenic effects are caused by drugs that have the capacity to cross the placenta and produce defects in the developing fetus. Some antibiotics may cause teratogenic effects. These can result in various congenital abnormalities. Carcinogenic drugs can cause cancer. Drugs with hallucinogenic effects cause hallucinations. Allergic effects are the result of allergies.

Why are infants especially sensitive to drugs that affect central nervous system (CNS) function? 1 The blood-brain barrier is especially strong in infants. 2 The blood-brain barrier is not fully developed at birth. 3 The blood-brain barrier does not exist until 1 year of age. 4 The blood-brain barrier is weakened by the birth process.

2) The blood-brain barrier is not fully developed at birth. The blood-brain barrier is not fully developed at birth. The other statements are not true.

A patient develops nausea, joint pain, weakness, and fatigue after suddenly stopping prednisone that has been taken for several months. What problem will the nurse suspect based on this new development? 1 The patient has rheumatoid arthritis. 2 The patient has adrenal insufficiency. 3 The patient has Cushing's syndrome. 4 The patient has compartment syndrome.

2) The patient has adrenal insufficiency. Prednisone is a glucocorticoid. If a systemic glucocorticoid is abruptly discontinued, adrenal insufficiency ensues. Nausea, joint pain, weakness, and fatigue are symptoms of adrenal insufficiency. If too much glucocorticoid is received or long-term glucocorticoid therapy is used, Cushing's syndrome can develop. Rheumatoid arthritis is an autoimmune disease, which is not caused by abrupt cessation of glucocorticoid therapy. Coronary ischemia causes shortness of breath, fatigue, and weakness but is not associated with discontinuation of glucocorticoid therapy.

The provider has prescribed a 7-day treatment of ciprofloxacin/dexamethasone ear drops for a patient with otitis externa. The nurse knows the combination medication has both antibacterial medication and topical glucocorticoid medication. Which major goal will the nurse add to the care plan for the glucocorticoid component? 1 To treat infection 2 To reduce inflammation 3 To prevent osteoporosis 4 To decrease the amount of cerumen

2) To reduce inflammation The purpose of a glucocorticoid is to assist in decreasing inflammation in the ear. Glucocorticoids do not treat the infection; that is a goal for antibacterial medication. Glucocorticoids increase the risk of osteoporosis when given over the long term; this is a short-term medication (7 days). Glucocorticoids do not affect earwax.

A patient has been taking docusate sodium [Colace] daily for 1 year. Which statement by the patient would indicate a complication associated with use of this drug? 1 "My doctor says that I've developed colon polyps." 2 "I've noticed that I'm having tremors now in my left hand." 3 "I only have a bowel movement when I take the medicine." 4 "The dental hygienist said I was losing the enamel on my teeth."

3) "I only have a bowel movement when I take the medicine." Docusate sodium is a surfactant laxative that softens stool by allowing water penetration. Chronic exposure to laxatives can diminish defecation reflexes, leading to further reliance on laxatives. Patient education is the key to reducing laxative abuse. Colon polyps, loss of tooth enamel, and tremors are unrelated to docusate sodium.

Which statement indicates that the patient needs additional teaching on oral hypoglycemic agents? 1 "I will monitor my blood sugar daily." 2 "I will limit my alcohol consumption." 3 "I will take the medication only when I need it." 4 "I will report symptoms of fatigue and loss of appetite."

3) "I will take the medication only when I need it." Oral hypoglycemic agents must be taken on a daily scheduled basis to maintain euglycemia and prevent long-term complications of diabetes. When alcohol is ingested with certain oral hypoglycemic drugs, the hypoglycemic effect can be intensified. The patient may experience fatigue and loss of appetite as side effects of the medication, and these should be reported to the healthcare provider. The patient needs to closely monitor blood sugar.

The nurse is teaching a medication regimen to a patient who has been prescribed an oral daily dose of prednisone. How will the nurse instruct the patient to take this medication to ensure safe administration? 1 "Take with aspirin." 2 "Take late in the evening." 3 "Take in the early morning." 4 "Take with nonsteroidal anti-inflammatory drugs (NSAIDs)."

3) "Take in the early morning." As with alternate-day treatment, patients taking single daily doses should administer their medicine before 9:00 AM. Early morning administration is also helpful in that it mimics the burst of glucocorticoids normally released by the adrenal glands at dawn. Taking prednisone in the evening is not as effective. The risk of ulceration is increased by concurrent use of other ulcerogenic drugs, such as aspirin and other NSAIDs.

Which instruction should the nurse give a patient who is to take bismuth subsalicylate for eradication of H. pylori? 1 "Nausea and diarrhea are common side effects." 2 "Do not drink alcohol while taking this medication." 3 "The drug can cause your bowel movements to be black." 4 "Take the medication through a straw to prevent staining of your teeth."

3) "The drug can cause your bowel movements to be black." Bismuth may impart a harmless black coloration to the tongue and the stool, and patients should be forewarned of this effect. Nausea and diarrhea are the most common side effects of clarithromycin [Biaxin], which also is used to treat H. pylori. Alcohol must be avoided when the patient is treated with metronidazole [Flagyl] for H. pylori infection because it may cause a disulfiram-like reaction. Tetracycline, which is also highly active against H. pylori, can cause staining of developing teeth.

Which statement by a nursing student about insulin glargine [Lantus] indicates effective learning? 1 ""The duration of action of this insulin is about 8 to 10 hours." 2 "You can mix this insulin with Lente insulin to enhance its effects." 3 "You cannot mix this insulin in the same syringe with regular insulin." 4 "This insulin is injected just before meals because it is very fast acting."

3) "You cannot mix this insulin in the same syringe with regular insulin." Insulin glargine [Lantus] is a long-acting insulin with a duration of action up to 24 hours. Hence, it can be given at bedtime to provide up to 24 hours of insulin coverage. It should not be mixed with any other insulin. It is given alone because of the low pH of the diluent.

A patient is receiving glucocorticoids for the treatment of rheumatoid arthritis. The patient complains of having a headache. Which ordered medication should the nurse administer? 1 Aspirin [Bayer] 2 Ibuprofen [Advil] 3 Acetaminophen [Tylenol] 4 Naproxen sodium [Aleve]

3) Acetaminophen [Tylenol] The risk of gastrointestinal irritation and ulceration for a patient taking glucocorticoids is increased by concurrent use of other medications, including aspirin [Bayer] and nonsteroidal anti-inflammatory drugs such as ibuprofen [Advil] and naproxen sodium [Aleve].

Follicle-stimulating hormone (FSH) and luteinizing hormone (LH) are produced by which structure? 1 Ovaries 2 Hypothalamus 3 Anterior pituitary gland 4 Posterior pituitary gland

3) Anterior pituitary gland FSH and LH are secreted by the anterior pituitary gland. They both act on the ovaries to promote either follicular growth and development or ovulation and the development of the corpus luteum. They act in a negative feedback loop with the anterior pituitary gland and hypothalamus to affect the ovaries.

A patient is prescribed an aluminum-containing antacid for hyperacidity. The nurse should inform the patient about which possible adverse effect? 1 Diarrhea 2 Flatulence 3 Constipation 4 Muscle twitching

3) Constipation Formulations containing aluminum or calcium cause constipation. Aluminum, used to reduce gastric acid, binds to phosphate and may lead to hypercalcemia. Early hypercalcemia is characterized by constipation. Diarrhea is an adverse effect of magnesium-containing antacids. Flatulence is due to the accumulation of gas in the intestine and is not related to the intake of aluminum-containing antacids. Muscle twitching is due to metabolic alkalosis resulting from the excessive use of sodium bicarbonate antacids.

Which statement about drug use among older adults is true? 1 Older adults require higher doses of most drugs. 2 Older adults experience fewer adverse drug reactions. 3 Drug use among older adults is disproportionately high. 4 Older patients are less sensitive to drugs than younger adults.

3) Drug use among older adults is disproportionately high. Drug use among older adults is disproportionately high. Older adults consume 33% of the nation's prescribed drugs. Older patients are more sensitive to drugs than are younger adults. Older adults experience more adverse drug reactions.

A patient is taking a glucocorticoid and a loop diuretic. The patient reports muscle weakness and cramping. Which assessment is priority? 1 Temperature 2 Stool for occult blood 3 Pulse rate and rhythm 4 Level of serum sodium

3) Pulse rate and rhythm Muscle weakness and cramping are indications the patient has hypokalemia from the combination of a glucocorticoid and a loop diuretic. Checking the pulse rate and rhythm is priority because cardiotoxicity can occur. Temperature is priority to detect an infection. Stool is tested for occult blood in case of suspected bleeding. Serum potassium is a priority, rather than serum sodium.

What should the nurse assess in a patient who is prescribed metformin [Glucophage] for treatment of type 2 diabetes? 1 Headache 2 Weight gain 3 Renal function 4 Cholesterol level

3) Renal function As metformin [Glucophage] is excreted by the kidneys, it is necessary to assess the patient's renal function. If the patient's kidneys are not able to excrete the drug, it will accumulate in the patient's system, thereby causing lactic acidosis. One of the adverse effects of metformin [Glucophage] is weight loss, not weight gain. Headaches are not caused by metformin [Glucophage]. Cholesterol levels may be high in some diabetic patients but can be treated with medications and lifestyle changes.

The provider writes a prescription for lorcaserin [Belviq]. Which medication on the patient's current medication list would prompt the nurse to provide further teaching to the patient? 1 Vitamin D 2 Meloxicam 3 Sertraline [Zoloft] 4 Omeprazole [Prilosec]

3) Sertraline [Zoloft] Serotonin syndrome is a risk associated with serotonergic drugs. When serotonergic drugs such as lorcaserin are given with other serotonergic drugs, this risk increases. Caution and close monitoring are advised when administering this drug along with selective serotonin reuptake inhibitors such as sertraline. Patients should be taught to monitor for symptoms of serotonin syndrome. Vitamin D, meloxicam, and omeprazole do not pose the same concerns.

Which hormone inhibits the release of growth hormone? 1 Somatropin 2 Somatolactin 3 Somatostatin 4 Choriomammotropin

3) Somatostatin The hormone that inhibits growth is somatostatin. It is otherwise called growth hormone inhibitor and is released from the digestive system and brain. Somatolactin is a hormone that induces oxytocin. Somatropin is a growth-stimulating hormone. Choriomammotropin belongs to the family of somatropin.

The nurse is assessing a patient who is on long-term glucocorticoid therapy. Following the assessment, the nurse finds that the patient has a "moon face," has excess fat deposited in the abdomen and trunk, and has a "buffalo hump." How will the nurse interpret these findings? 1 The patient has hypokalemia. 2 The patient has adrenal insufficiency. 3 The patient has iatrogenic Cushing's syndrome. 4 The patient has a fluid and electrolyte disturbance.

3) The patient has iatrogenic Cushing's syndrome. Presence of a "moon face," excess fat deposition in the abdomen and trunk, and "buffalo hump" indicate iatrogenic Cushing's syndrome. A fluid and electrolyte disturbance results in sodium and water retention and potassium loss (hypokalemia). Adrenal insufficiency results in inability to handle stress and has symptoms of hypotension, hypoglycemia, myalgia, arthralgia, and fatigue.

A patient with a chronic illness is on daily glucocorticoids. He is brought to the hospital following a severe motor vehicle accident. He is wearing a medical alert bracelet stating his home medications. Noting that the patient takes glucocorticoids daily, the nurse knows which of the following to be true? 1 The patient will have to stop all glucocorticoids immediately. 2 The patient will need the same dose of glucocorticoids he used at home. 3 The patient will need higher doses of glucocorticoids following the trauma. 4 The patient will need lower doses of glucocorticoids following the trauma.

3) The patient will need higher doses of glucocorticoids following the trauma. During times of stress, the adrenal glands normally secrete large amounts of glucocorticoids. If the stress is sufficiently severe (eg, trauma, surgery), these glucocorticoids are essential for supporting life. Accordingly, because of adrenal suppression, it is imperative that patients receiving long-term glucocorticoid therapy be given increased doses at times of stress.

A nurse is teaching a patient about regulation and secretion of glucocorticoids. In which order should the nurse list the steps? 1. Anterior pituitary stimulated 2. Adrenal cortex stimulated 3. Hypothalamus stimulated 4. Glucocorticoids released 5. Adrenocorticotropic hormone (ACTH) released 6. Corticotrophin-releasing hormone (CRH) released

3. Hypothalamus stimulated 6. Corticotrophin-releasing hormone (CRH) released 1. Anterior pituitary stimulated 5. Adrenocorticotropic hormone (ACTH) released 2. Adrenal cortex stimulated 4. Glucocorticoids released The steps are as follows: the hypothalamus is stimulated from stress or other central nervous system stimuli; then occurs the release of corticotrophin-releasing hormone (CRH); this leads to anterior pituitary stimulation, releasing adrenocorticotropic hormone (ACTH), which stimulates the adrenal cortex to release glucocorticoids.

The healthcare provider ordered thiamine solution 100 mg in 50 mL IV piggyback for a patient with a history of alcohol abuse. The dose is ordered to run at 100 mL per hour. The nurse knows that the infusion will require how much time? Record your answer using a whole number. _____ minutes

30 The medication is 100 mg in 50 mL and is ordered to run at 100 mL per hour. Since the medication is only 50 mL, the infusion would be complete in 30 minutes.

The nurse is providing education to a group of patients interested in complementary medicine. Which teaching point should the nurse include as priority education for the group? 1 "Understand the use of any herb before taking it." 2 "Stop taking any herb if you note any adverse effects." 3 "Read the directions and labels of all herbs before taking." 4 "Confirm with your healthcare provider that any herbs you take will not interact with prescribed medications."

4) "Confirm with your healthcare provider that any herbs you take will not interact with prescribed medications." The highest priority teaching point is that the patient should seek education and confirm with a healthcare provider that any herbs taken will not have an adverse effect if taken with prescribed medications.

The nurse is teaching a patient receiving long-term glucocorticoid drug therapy. Which response by the patient indicates the need for follow-up by the nurse? 1 "I will take this medication before 9 AM." 2 "I will not touch or interact with people who have infections." 3 "I will report to you immediately if I have fever or a sore throat." 4 "I will stop taking this medication immediately if I develop an infection."

4) "I will stop taking this medication immediately if I develop an infection." The nurse should follow up if the patient indicates abrupt stoppage of medication, as this is a misconception that must be corrected. Abrupt termination of long-term therapy may unmask adrenal insufficiency. To minimize the impact of adrenal insufficiency, glucocorticoid withdrawal should be gradual. Patients must be warned against abrupt discontinuation. All the other statements are correct. Fever and a sore throat indicate possible infection, and the patient should contact the nurse or provider immediately if they occur. Because these medications suppress the immune system, patients taking corticosteroids need to avoid contact with people who have an infection. Glucocorticoids are to be taken before 9 AM.

A 3-year-old child has been started on a new medication to be given as needed. What is the most important information to convey to the parents? 1 "Make sure the child likes the taste of the medication." 2 "Mix the medication with the child's favorite food or beverage." 3 "Make sure you give the medication at the same time every day." 4 "Observe the child for potential adverse effects of the medication."

4) "Observe the child for potential adverse effects of the medication." Adverse effects of medications can be dangerous as well as difficult to discern in young children, especially effects such as ringing in the ears, because the child might not be able to communicate this symptom. Parents are in the best position to observe the child and note changes in behaviors that might be related to side effects. Drugs that are ordered to be given as needed should not be given routinely on a schedule. Not all medications can be made palatable. The nurse can work with parents to help with successful medication administration even with medications that do not taste good.

The nurse is teaching safe administration of medication to a patient who has been prescribed levothyroxine [Levoxyl]. Which statement should the nurse include in the teaching session? 1 "Take this medication with 250 mL of orange juice." 2 "Always take the medication three times a day, after meals." 3 "There are no dietary restrictions while taking this medication." 4 "Take this medicine on an empty stomach in the morning, at least 30 to 60 minutes before eating."

4) "Take this medicine on an empty stomach in the morning, at least 30 to 60 minutes before eating." Patients who are prescribed thyroid replacements or antithyroid drugs should be advised to take the medicine on an empty stomach in the morning, at least 30 to 60 minutes before eating. This helps enhance the absorption of the drug. Taking the medication twice a day after meals may reduce the therapeutic effectiveness of the medication and cause adverse effects. A patient needs to avoid eating foods that may reduce thyroid hormone production and reduce the effectiveness of the medication. Therefore, the nurse should not give false information that the patient need not follow dietary restrictions. This medication should be taken with water rather than orange juice, as it helps enhance the disintegration and absorption of the drug.

Which statement is the most important for a nurse to make to a patient who is taking methimazole [Tapazole]? 1 "Headache and dizziness may occur but not very frequently." 2 "Another medication can be given if you experience any nausea." 3 "You may experience some muscle soreness with this medicine." 4 "You need to notify your doctor if you have a sore throat and fever."

4) "You need to notify your doctor if you have a sore throat and fever." Agranulocytosis (the absence of granulocytes to fight infection) is the most serious toxicity associated with methimazole. Sore throat and fever may be the earliest signs. Nausea, muscle soreness, headache, and dizziness are other adverse effects of methimazole that are less serious than agranulocytosis.

The primary healthcare provider prescribes a thyroid replacement drug to a patient with hypothyroidism. How should the nurse monitor for return to a euthyroid state? 1 Assess for anxiety and palpitations. 2 Verify the medication history of patient and family. 3 Evaluate the patient's knowledge of thyroid therapy. 4 Analyze the patient's serum thyroid-stimulating hormone levels.

4) Analyze the patient's serum thyroid-stimulating hormone levels. Drug efficacy is assessed by monitoring the thyroid-stimulating hormone [1] [2] (TSH) levels. The nurse will also monitor other thyroid tests, if ordered, and will assess for symptom improvement. Anxiety and palpitations would indicate a hyperthyroid state, which could occur with drug accumulation or excess. The other items, while important, do not address monitoring for a euthyroid (normal) state.

A patient reports abdominal bloating and infrequent, small, hard stools after taking psyllium [Metamucil] for 2 weeks. Which is the nurse's priority action? 1 Consult the physician about another laxative choice. 2 Check the dose because an increase may be indicated. 3 Ask whether the patient is toileting at the same time every day. 4 Ask whether the patient is using at least 8 ounces of fluid to prepare the psyllium.

4) Ask whether the patient is using at least 8 ounces of fluid to prepare the psyllium. Bulk-forming laxatives, such as psyllium, must be given with at least 8 ounces (240 mL) of liquid, plus additional liquid each day, to prevent intestinal impaction. Another laxative may not be necessary at this time. A dosage increase and monitoring are appropriate after proper mixing of the medication has been validated.

A nurse is planning care for a patient who has peptic ulcer disease and is taking amoxicillin [Amoxil]. The nurse is aware that amoxicillin has which mechanism of action? 1 Coating of the ulcer crater as a barrier to acid 2 Inhibition of an enzyme to block acid secretion 3 Selective blockade of parietal cell histamine2 receptors 4 Disruption of the bacterial cell wall, causing lysis and death

4) Disruption of the bacterial cell wall, causing lysis and death Amoxicillin disrupts the cell wall of H. pylori, which causes lysis and death. Inhibition of an enzyme to block acid secretion is a function of the proton pump inhibitors (PPIs). Coating of the ulcer crater as a barrier to acid is an action of sucralfate [Carafate]. Selective blockade of parietal cell histamine2 receptors is an action of the histamine2 receptor antagonists cimetidine, ranitidine, famotidine, and nizatidine.

When calculating pediatric dosages, what should the nurse take into consideration? 1 Utilization of drug reference recommendations based on mg/kg of body weight is the preferred method. 2 Calculated doses based on body weight should be increased by 10% because of immature renal and hepatic function. 3 Dosage calculation according to body weight is the most accurate method because it takes into account differences in maturational development. 4 Dosage calculation by body surface area is the most accurate method because it takes into account the difference in size of the child and/or neonate.

4) Dosage calculation by body surface area is the most accurate method because it takes into account the difference in size of the child and/or neonate. The body surface area takes into account not only the child's weight but also the relationship with height and is therefore both the most accurate and most preferred method. Immature renal and hepatic function would necessitate a decrease in dose, not an increase.

The provider has prescribed intravenous administration of promethazine [Phenergan]. The nurse would question the intravenous (IV) use instead of intramuscular (IM) administration of promethazine [Phenergan] for which reason? 1 IM injection has a more rapid onset of action. 2 The risk of respiratory depression is eliminated with IM injection. 3 Extrapyramidal reactions do not occur when the drug is administered IM. 4 Extravasation of IV promethazine can lead to abscess formation or tissue necrosis.

4) Extravasation of IV promethazine can lead to abscess formation or tissue necrosis. Extravasation of IV promethazine can lead to abscess formation, tissue necrosis, and gangrene, leading to amputation; therefore, IV administration should be avoided. If it must be done, promethazine should be given through a large-bore IV line. Respiratory depression and extrapyramidal side effects can occur regardless of the route of administration. Drugs administered IM have a longer onset of action than those given IV.

Which condition is seen in a patient with hyperosmolar nonketotic syndrome (HHNS)? 1 Acute pancreatitis 2 Recurrent headaches 3 Insidious weight gain 4 Extreme hyperglycemia

4) Extreme hyperglycemia Extreme hyperglycemia triggers hyperosmolar nonketotic syndrome (HHNS). Acute pancreatitis is an inflammation of the pancreas. This usually happens due to chronic alcohol abuse or due to a gallstone becoming lodged in the pancreatic ducts. Headaches are not caused by HHNS. A patient with HHNS may have electrolyte imbalances, which may cause dehydration and weight loss, not weight gain.

A patient has been taking aluminum hydroxide [Amphojel] to treat gastric hyperacidity for a few days. The patient reports being constipated. Which drug will the provider likely order in addition to aluminum hydroxide [Amphojel]? 1 Sodium citrate [Citra pH] 2 Calcium carbonate [Tums] 3 Aluminum carbonate [Basaljel] 4 Magnesium hydroxide [Milk of Magnesia]

4) Magnesium hydroxide [Milk of Magnesia] The constipation caused by the aluminum-based antacid is neutralized when taken in combination with a magnesium-based antacid such as magnesium hydroxide [Milk of Magnesia], which acts as a laxative. Administration of aluminum carbonate [Basaljel] and aluminum hydroxide [Amphojel] would aggravate constipation. Sodium-based antacids such as sodium citrate [Citra pH] along with aluminum hydroxide [Amphojel] have no effect in relieving the effects of the latter. Calcium-based antacids such as calcium carbonate [Tums] also cause constipation, and hence their use will not alleviate the patient's discomfort.

A patient with type 1 diabetes mellitus has been ordered insulin aspart [NovoLog] 10 units at 7:00 AM. Which nursing intervention should the nurse perform after administering this medication? 1 Flush the intravenous line. 2 Perform a fingerstick blood sugar test. 3 Have the patient void and dipstick the urine. 4 Make sure the patient eats breakfast immediately.

4) Make sure the patient eats breakfast immediately. Insulin aspart [NovoLog] is a rapid-acting insulin that acts in 15 minutes or less. It is imperative that the patient eat as the insulin starts to work. The patient should have had a fingerstick blood sugar test done before receiving the medication. There is no need to check the urine. This medication is given subcutaneously.

A patient who has depression is admitted to the emergency department with serotonin syndrome. The nurse learns that the patient is taking an herbal medication and citalopram [Celexa]. Which herbal medication is the patient most likely taking? 1 Valerian 2 Cranberry 3 Saw palmetto 4 St. John's wort

4) St. John's wort St. John's wort is an herb used as a medication to treat depression. It leads to serotonin syndrome when administered with other selective serotonin reuptake inhibitors, such as citalopram [Celexa]. Valerian causes central nervous system depression when used with sedatives. Saw palmetto changes the effects of hormones in oral contraceptive drugs or hormonal replacement therapies. Cranberry decreases the elimination of many medications excreted by the kidneys.

The endocrinologist orders lanreotide [Somatuline Depot] for a patient with acromegaly. Which should the nurse identify as the reason for prescribing this drug? 1 Relief of bone pain 2 Prevention of osteopenia 3 Promotion of epiphyseal closure 4 Suppression of growth hormone release

4) Suppression of growth hormone release The somatostatin analogs—octreotide and lanreotide—are the most effective drugs for suppressing growth hormone (GH) release. Benefits derive from mimicking the suppressant actions of somatostatin on the pituitary (see Fig. 59-3 in the text). The somatostatin analogs can be used as primary therapy for acromegaly or as an adjunct to surgery and/or radiation. In both cases, the objective is to normalize levels of GH and insulin-like growth factor-1.

Which manifestations should a nurse investigate first when monitoring a patient who is taking levothyroxine [Synthroid]? 1 Tremors 2 Insomnia 3 Irritability 4 Tachycardia

4) Tachycardia High doses of levothyroxine may cause thyrotoxicosis, a condition of profound, excessive thyroid activity. Tachycardia is the priority assessment because it can lead to severe cardiac dysfunction. Tremors, insomnia, and irritability are other symptoms of thyrotoxicosis and should be assessed after tachycardia.

A female patient is given a prescription for lorcaserin [Belviq]. The nurse ensures that the patient schedules an appointment with which type of healthcare provider? 1 Oncologist 2 Dermatologist 3 Gastroenterologist 4 Women's healthcare provider or gynecologist

4) Women's healthcare provider or gynecologist A female patient on lorcaserin should see her women's health or gynecology provider for birth control recommendations. Lorcaserin is classified as Pregnancy Risk Category X. Women who are pregnant or who are at risk of becoming pregnant should not take this drug. Breast-feeding, although not contraindicated, is not recommended for women taking this drug. Other healthcare providers are not specifically indicated for the use of this medication.

A patient is going to begin glucocorticoid therapy. The nurse should educate the patient about which complication(s)? Select all that apply. A Predisposition to gastrointestinal ulceration B Impaired carbohydrate and glucose tolerance C Decrease in serum sodium and glucose levels D Increase in plaque development in coronary arteries E Increased production of glucocorticoids from the adrenals

A Predisposition to gastrointestinal ulceration B Impaired carbohydrate and glucose tolerance Adverse effects of glucocorticoids occur in response to pharmacologic doses. They include suppression of adrenal glucocorticoids, glucose and carbohydrate intolerance, elevated blood glucose level, water and sodium retention, and increased gastric acid secretion. Plaque development in arteries is not affected.

A patient with systemic lupus erythematosus receives glucocorticoids. Which adverse effects will the nurse monitor for in this patient? Select all that apply. A Weight gain B Hypoglycemia C Increased sleep D Personality changes E Loss of bone density F Loss of muscle mass

A Weight gain D Personality changes E Loss of bone density F Loss of muscle mass The patient is at high risk for osteoporosis as a result of glucocorticoid therapy because glucocorticoids are associated with bone demineralization and loss of bone density. Weight gain (from fluid retention) and personality changes also are associated with glucocorticoid therapy. Loss of muscle mass can also be an adverse effect of glucocorticoid therapy. Glucocorticoid therapy can cause insomnia and hyperglycemia, not increased sleep or hypoglycemia.

Which discharge information will the nurse share with the patient who will be taking long-term glucocorticoid therapy? Select all that apply. A "Avoid crowds." B "Restrict calcium intake." C "Increase sodium intake." D "Wear an identification bracelet." E "Keep an emergency supply of glucocorticoids on hand."

A) "Avoid crowds." D) "Wear an identification bracelet." E) "Keep an emergency supply of glucocorticoids on hand." To ensure appropriate care in emergencies, patients should carry an identification card or bracelet to inform emergency personnel of their glucocorticoid needs. In addition, patients should always have an emergency supply of glucocorticoids on hand. By suppressing host defenses (immune responses and phagocytic activity of neutrophils and macrophages), glucocorticoids can increase susceptibility to infection and the patient should avoid crowds. Calcium should be increased and sodium decreased.

The nurse is teaching the client about dietary factors that increase risks for colon cancer. The most important information for the nurse to include would be: Select all that apply. a) "Decrease your daily intake of vitamins A and E." b) "Decrease the amount of fiber in your diet to decrease the bulk." c) "Increase your daily amount of fat consumption." d) "Increase daily amounts of vitamin C." e) "Increasing your daily amount of sugar intake does influence your risk."

a) "Decrease your daily intake of vitamins A and E." e) "Increasing your daily amount of sugar intake does influence your risk." Attention has focused on dietary fat intake, refined sugar intake, fiber intake, and the adequacy of such protective micronutrients as vitamins A, C, and E in the diet. A high level of fat in the diet increases the synthesis of bile acids in the liver, which may be converted to potential carcinogens by the bacterial flora in the colon. The proliferation of colonic bacteria is enhanced by a high dietary level of refined sugars. Dietary fiber is thought to increase stool bulk and thereby dilute and remove potential carcinogens. Refined diets often contain reduced amounts of vitamins A, C, and E, which may act as oxygen free radical scavengers.

A client diagnosed with type 2 diabetes has been instructed about managing his condition with diet. The nurse determines further teaching is necessary when the client states: a) "I must avoid all candies and cookies, but can eat unlimited amounts of pasta and breads." b) "I need to avoid adding salt to my foods." c) "I can drink 8 to 10 glasses of water daily without concern for calories." d) "I need to limit the amount of foods that I eat that contain trans fats."

a) "I must avoid all candies and cookies, but can eat unlimited amounts of pasta and breads." The client needs more education regarding carbohydrate sources. All the other statements are correctly stated and demonstrate good understanding of dietary needs for the type 2 diabetic.

When providing nutrition education to the client with diabetes, the nurse should include which of these statements regarding fat intake? a) "If you have diabetes, avoiding saturated fats is important." b) "Individuals with type 2 diabetes may consume more fat calories than those with type 1 diabetes." c) "People with diabetes should not consume fats." d) "When diabetes is present, 40% of calories may be from fat."

a) "If you have diabetes, avoiding saturated fats is important." Guidelines for caloric intake for those with diabetes may include 45% to 60% carbohydrate, 20% to 35% fat, and 10% to 20% protein. Because diabetes is a risk factor for cardiovascular disease, it is recommended that less than 7% of daily calories should be obtained from saturated fat and that dietary cholesterol be limited to 200 mg or less, and intake of trans fats minimized.

A student is comparing the different parts of the gastrointestinal (GI) tract. Which of the following statements demonstrates understanding? a) "The mouth is the receptacle where initial digestive processes take place." b) "The cecum is where most digestive and absorptive processes occur." c) "The jejunum serves as a storage channel for the elimination of waste." d) "The esophagus is where most of the absorptive processes occur."

a) "The mouth is the receptacle where initial digestive processes take place." The GI tract can be divided into three parts: upper, middle, and lower segments. The upper part contains the mouth and esophagus and acts as a receptacle through which food passes and initial digestive processes take place. The middle portion, containing the jejunum, is where most digestive and absorptive processes occur. The lower segment, containing the cecum, serves as a storage channel for the efficient elimination of waste.

A patient takes 650 mg of aspirin every 4 hours daily for complaints of joint pain. Which of the following statements should be included in the patient's teaching plan? a) "This medication can damage gastric mucosa." b) "You should have your hemoglobin and hematocrit checked weekly." c) "This medication will not assist with joint pain." d) "You should have your partial thromboplastin time assessed."

a) "This medication can damage gastric mucosa." Aspirin and nonsteroidal anti-inflammatory drugs can damage the gastric mucosa. Partial thromboplastin is not affected, nor is hemoglobin or hematocrit. The medication should help to decrease inflammation in the joints.

A client with Graves' disease has opthalmopathy and asks the nurse if the eyes will stay like this forever. What is the best response by the nurse? a) "With treatment of the hyperthyroid state, the opthalmopathy usually tends to stabilize." b) "Once we treat your Graves' disease, your eyes will go back to their normal state." c) "The protrusion of the eyes will get worse before they get better." d) "Your eyes will be like this but there are things we can do to reduce visual loss."

a) "With treatment of the hyperthyroid state, the opthalmopathy usually tends to stabilize." The ophlalmopathy of Graves' disease can cause severe eye problems, including tethering of the extraocular muscles resulting in diplopia; involvement of the optic nerve, with some visual loss; and corneal ulceration because the lids do not close over the protruding eyeball. The opthalmopathy usually tends to stabilize after treatment of the hyperthyroidism.

The nurse is working in a pediatric clinic. Which of the following children would the nurse recognize as having isosexual precocious puberty? a) A 5-year-old African-American female with developing breasts and pubic hair b) A 9-year-old Caucasian female who has small breast buds c) A 14-year old Caucasian female who has not yet developed secondary sex characteristics d) A 10-year-old African-American female who has begun menstruating

a) A 5-year-old African-American female with developing breasts and pubic hair Precocious puberty is now defined as the appearance of secondary sexual development before the age of 7 years in white girls and 6 years in African-American girls. In boys of both races, the lower age limit remains 9 years. However, it is recognized that puberty can develop earlier in boys with obesity.

Select the client at greatest risk for developing colorectal cancer. a) A 64-year-old female whose mother had colorectal cancer b) A 45-year-old female who takes four to six aspirin per week for arthritis c) A 26-year-old male with a history of irritable bowel syndrome d) A 40-year-old male with a history of peptic ulcer disease

a) A 64-year-old female whose mother had colorectal cancer Colorectal cancer peaks at 60 to 70 years of age, and fewer than 20% of cases occur before age 50. Its incidence is increased among persons with a family history of cancer, persons with Crohn disease or ulcerative colitis, and those with familial adenomatous polyposis of the colon. Aspirin or other NSAIDs may protect against colorectal cancer. IBS and peptic ulcers are not risk factors

Which of the following individuals most likely faces the greatest risk of developing Clostridium difficile colitis? a) A 79-year-old hospital client who is being treated with broad-spectrum antibiotics b) A 55-year-old man who takes proton pump inhibitors for the treatment of peptic ulcers c) A 30-year-old client who has a history of Crohn disease and has been admitted to a hospital to treat a recent flare-up d) A premature neonate who has developed hyperbilirubinemia and is receiving phototherapy

a) A 79-year-old hospital client who is being treated with broad-spectrum antibiotics C. difficile colitis is associated with antibiotic therapy; C. difficile is noninvasive, and development of C. difficile colitis requires disruption of normal intestinal flora. Peptic ulcers, hyperbilirubinemia, and Crohn disease are not common risk factors for the development of C. difficile colitis.

Which of the following clients likely faces the greatest risk of a gastrointestinal bleed? a) A client who takes aspirin with each meal to control symptoms of osteoarthritis b) A man whose hypertension requires him to take a diuretic, an ACE inhibitor, and a β-adrenergic blocker c) A client who is taking a broad-spectrum antibiotic to treat a urinary tract infection d) A client with a history of anxiety who takes benzodiazepines several times daily

a) A client who takes aspirin with each meal to control symptoms of osteoarthritis The gastric mucosa can be easily damaged by drugs such as aspirin, resulting in local ischemia, vascular stasis, hypoxia, and tissue necrosis. Antihypertensives, diuretics, antibiotics, and benzodiazepines do not pose such a significant threat to the integrity of the gastric mucosa

When explaining factors that influence the number of receptors present on target cells, the instructor will likely mention: Select all that apply. a) A decreased hormone level may produce increased receptor numbers. b) The role antibodies may have on receptor proteins. c) A sustained excess hormone level brings about a decrease in receptor numbers. d) It may take days to weeks before a hormone can react to target cells. e) How any given hormone can change its affinity to supply a need to all cells.

a) A decreased hormone level may produce increased receptor numbers. b) The role antibodies may have on receptor proteins. c) A sustained excess hormone level brings about a decrease in receptor numbers. Target cell response varies with the number and affinity of the relevant receptors. The number of hormone receptors on a cell may be altered for any of several reasons. Antibodies may destroy or block the receptor proteins. Increased or decreased hormone levels often induce changes in the activity of the genes that regulate receptor synthesis. For example, decreased hormone levels often produce an increase in receptor numbers by means of a process called up-regulation; this increases the sensitivity of the body to existing hormone levels. Likewise, sustained levels of excess hormone often bring about a decrease in receptor numbers by down-regulation, producing a decrease in hormone sensitivity.

When caring for a client with hypomagnesemia, the nurse prioritizes assessment of which of these systems? a) Cardiac b) Pulmonary c) Hepatic d) Endocrine

a) Cardiac Assessment of the cardiac system is essential as magnesium is needed for proper nervous system function and to maintain the tone of the blood vessels. Low magnesium levels produce an increase in irritability of the heart causing cardiac arrhythmias.

A nurse is concerned about a patient's continual use of nonsteroidal anti-inflammatory drugs (NSAIDs). Which of the following is the nurse concerned about? a) Damage to the gastric mucosa b) Increase in the proton pump c) Increase in gastrin secretion d) Decrease in the production of intrinsic factor

a) Damage to the gastric mucosa One of the important characteristics of the gastric mucosa is resistance to highly acidic secretions that it produces. However, when aspirin, NSAIDs, Heliocbacter pylori, ethyl alcohol, or bile salts damage the gastric mucosa, this barrier is disrupted. NSAIDs do not increase the production of gastric acid (proton pump), decrease the production of intrinsic factor, or increase the secretion of gastrin.

A nurse caring for a patient who is vomiting assesses for accompanying symptoms including which of the following? Select all that apply. a) Decreased blood pressure b) Dizziness c) Increased pulse d) Lightheadedness

a) Decreased blood pressure b) Dizziness Vomiting may be accompanied by dizziness, lightheadedness, a decrease in blood pressure, and bradycardia.

A nurse is conducting a health promotion class on healthy cooking. The nurse includes the importance of ensuring that all hamburgers are cooked completely before ingesting to prevent which of the following? a) Escherichia coli b) Rotovirus c) Entamoeba histolytica d) Clostridium difficile

a) Escherichia coli E. coli is a strain of E. coli found in the feces and contaminated milk of healthy diary and beef cattle. Infection is by food-borne transmission, often by ingesting undercooked hamburger.

Crohn's disease has a distinguishing pattern in the gastrointestinal (GI) tract. The surface has granulomatous lesions surrounded by normal-appearing mucosal tissue. A complication of the pattern includes which of the following? a) Fistula formation b) Dysphagia c) Rectal bleeding d) Constipation

a) Fistula formation In Crohn's disease all layers of the bowel are involved. Complications of Crohn's disease include fistula formation, abdominal abscess formation, and intestinal obstruction. Fistulas are tubelike passages that form connections between different sites in the GI tract.

Which measure would likely be rejected as part of a first-line weight loss plan for a client with a BMI of 30.2, type 2 diabetes, and hypertension? a) Gastric bypass b) Prescription-drug therapy c) Calorie reduction by 500 to 1,000 kcal/day d) 30 minutes or more of moderate-intensity activity at least 3 days per week

a) Gastric bypass Weight-loss surgery should be limited to individuals with a BMI greater than 40 or those with a BMI greater than 35 with comorbid conditions in whom medical therapy has failed. Calorie reduction is appropriate for anyone with a BMI of 25 to 29.9 plus two risk factors (in this case, type 2 diabetes and hypertension). Prescription-drug therapy can be considered for those with a BMI of 30 or more, and increased physical activity, although it does not lead to significant weight loss, helps prevent further weight gain and reduces cardiovascular and diabetes risk beyond that achieved by weight loss alone.

Type 1A diabetes is now considered an autoimmune disorder. What factors are considered necessary for type 1A diabetes to occur? a) Genetic predisposition, environmental triggering event, and a T-lymphocyte-mediated hypersensitivity reaction against some beta-cell antigen b) Diabetogenic gene from both parents, environmental triggering event, and a B lymphocyte reaction to alpha cell antigens c) Genetic predisposition, physiologic triggering event, allergic reaction to pancreatic alpha cells d) Diabetogenic gene from both parents, physiologic triggering event, and an allergic reaction to pancreatic delta cells

a) Genetic predisposition, environmental triggering event, and a T-lymphocyte-mediated hypersensitivity reaction against some beta-cell antigen Type 1A diabetes is thought to be an autoimmune disorder resulting from a genetic predisposition (i.e., diabetogenic genes); an environmental triggering event, such as an infection; and a T-lymphocyte-mediated hypersensitivity reaction against some beta-cell antigen. The other answers are incorrect.

Which of the following hormones is produced by the anterior pituitary gland? a) Growth hormone (GH) b) Corticotropin-releasing hormone (CRH) c) Norepinephrine d) Oxytocin

a) Growth hormone (GH) GH is among the hormones produced and released by the anterior pituitary. Oxytocin is a posterior pituitary hormone, whereas CRH is produced by the hypothalamus. Norepinephrine and epinephrine are produced by the adrenal medulla.

In collecting assessment data on the school-aged population, which of the following factors could be the most significant predictor of childhood obesity? a) Having parents who are obese b) Low self-esteem c) Low socioeconomic status d) Living in a rural neighborhood

a) Having parents who are obese The most significant risk for childhood obesity is having parents who are obese. This variable is more important than socioeconomic status, low self-esteem, or a rural or inner-city residence.

A patient is diagnosed with Crohn's disease. The nurse instructs the patient on which of the following types of dietary needs? a) High-calorie, vitamin, and protein diet b) High-fat diet, low calorie c) High-residue diet d) High-fruit and vegetable intake

a) High-calorie, vitamin, and protein diet Nutritional deficiencies are common in Crohn's disease because of diarrhea, steatorrhea, and other malabsorption problems. A nutritious diet that is high in calories, vitamins, and proteins is recommended. Because fats often aggravate the diarrhea, it is recommended they be avoided. Elemental diets, which are nutritionally balanced but residue free and bulk free, may be given during the acute phase of the illness.

During periods of fasting and starvation, the glucocorticoid and other corticosteroid hormones are critical for survival because of their stimulation of gluconeogenesis by the liver. When the glucocorticoid hormones remain elevated for extended periods of time, what can occur? a) Hyperglycemia b) Hepatomegaly c) Portal hypertension d) Adrenal hyperplasia

a) Hyperglycemia In predisposed persons, the prolonged elevation of glucocorticoid hormones can lead to hyperglycemia and the development of diabetes mellitus and starvation. They stimulate gluconeogenesis by the liver, sometimes producing a 6- to 10-fold increase in hepatic glucose production. A prolonged increase in glucocorticoid hormones does not cause hepatomegaly, portal hypertension, or adrenal hyperplasia.

A patient asks the nurse what part of the brain regulates appetite. Which is the best response by the nurse? a) Hypothalamus b) Cerebellum c) Medulla d) Midbrain

a) Hypothalamus Appetite or the desire for food is regulated by the hypothalamus and other associated parts of the brain. The medulla deals with the autonomic (involuntary) functions of breathing, heart rate, and blood pressure. The midbrain acts as a relay information system for the auditory, visual, and motor systems of the body. The cerebellum coordinates voluntary movements, posture, and balance in humans.

When the nurse is performing a health history for a client who is being admitted for hyperthyroidism, what symptoms does the client report that the nurse would find associated with this disorder? a) Increase in appetite b) Constipation c) Weight gain d) Fatigue

a) Increase in appetite Thyroid hormone enhances gastrointestinal function, causing an increase in motility and production of GI secretions that often results in diarrhea. An increase in appetite and food intake accompanies the higher metabolic rate that occurs with increased thyroid hormone levels. At the same time, weight loss occurs because of the increased use of calories.

A 20-year-old man who is addicted to methamphetamines has been admitted to hospital with a diagnosis of protein--calorie malnutrition after many months of inadequate food intake. Which treatment plan would the care team most likely favour? a) Incremental feeding combined with vitamin and mineral supplementation. b) Intravenous infusion of albumin coupled with vitamin supplementation. c) Total parenteral nutrition. d) Rapid administration of normal saline and carbohydrates.

a) Incremental feeding combined with vitamin and mineral supplementation. Slow administration of protein and calories combined with mineral and vitamin supplementation is important in the treatment of protein--calorie malnutrition. Albumin transfusions and total parenteral feeding would likely not be necessary because the client has no noted swallowing or metabolic problems, and rapid administration of fluids and carbohydrates may precipitate congestive heart failure.

Which of the following interventions is important for the nurse to teach a patient diagnosed with distal symmetric neuropathy related to diabetes? Select all that apply. a) Inspect the feet daily for blisters. b) Wear comfortable, open-toe shoes. c) Rotate insulin injection sites once a week. d) Decrease daily walking activity. e) Wear well-fitted shoes.

a) Inspect the feet daily for blisters. e) Wear well-fitted shoes. A patient with neuropathy is at risk for damage to their feet, such as blisters or ulcers, as they are unable to detect injuries. Patients need to inspect their feet daily, wear foot coverings (such as well-fitted closed-toe shoes) to prevent injuries, and continue exercise to improve circulation and glucose use. Insulin injection sites must be rotated for each administration.

Select the most appropriate intervention for the nurse to teach a client diagnosed with distal symmetric neuropathy related to diabetes. a) Inspect the feet for blisters daily b) Rotate insulin injection sites once a week c) Decrease daily walking activity d) Wear comfortable, open-toe shoes

a) Inspect the feet for blisters daily A client with neuropathy is at risk for damage to his or her feet, such as blisters or ulcers, as the clients are unable to feel this damage. Clients need to inspect their feet daily, where foot coverings (such as closed-toe shoes) to prevent injuries, and continue the exercise patterns to promote improved circulation.

The nurse has just completed teaching a client newly diagnosed with type 1 diabetes about rapid-acting insulin. The nurse determines that teaching was effective when the client selects: a) Lispro b) Regular c) NPH d) Glargine

a) Lispro There are four principal types of insulin: short acting, rapid acting, intermediate acting, and long acting. Regular insulin is short-acting insulin whose effects begin within 30 minutes after subcutaneous injection and generally last for 5 to 8 hours. The rapid-acting insulins (lispro, aspart, and glulisine) have a more rapid onset, peak, and duration of action than short-acting regular insulin and are administered immediately before a meal. Intermediate- to long-acting insulins include NPH, glargine, and detemir. These insulins have slower onsets and a longer duration of action.

When trying to explain to parents of school-aged children how to find the "good" fats that should used for cooking, which explanation is most accurate? a) Look for plant-based oils like "canola" on the label. b) You should see "saturated fatty acids" on the ingredient list. c) Look for "totally hydrogenated" fats on the list of ingredients. d) Look at the food label for "trans fatty acids."

a) Look for plant-based oils like "canola" on the label. The saturated fatty acids elevate blood cholesterol, whereas the monounsaturated and polyunsaturated fats lower blood cholesterol. Saturated fats are usually derived from animal sources and remain solid at room temperature. With the exception of coconut and palm oils (which are saturated), unsaturated fats are found in plant oils (like corn or canola)and usually are liquid at room temperature. Trans fatty acids are produced when unsaturated oils are partially hydrogenated and are called artificial trans fats. They are found primarily in vegetable shortenings and some margarines and foods.

The hormone levels in the body need to be kept within an appropriate range. How is this accomplished for many of the hormones in the body? a) Negative feedback loop b) Sensory feedback loop c) Positive feedback loop d) Regulated feedback loop

a) Negative feedback loop The level of many of the hormones in the body is regulated by negative feedback mechanisms. The other answers are incorrect.

An adult patient with suspected hypothyroidism is scheduled for a thyrotropin-releasing hormone (TRH) stimulation test to evaluate pituitary response. Which of the following test results would confirm secondary hypothyroidism? a) No increase in TSH b) No increase in calcitonin c) Increased calcitonin d) Increased thyroid-stimulating hormone (TSH)

a) No increase in TSH A stimulation test is intended to determine if an organ that is undersecreting will produce an increased response. A normal response to the TRH stimulation test would be for the pituitary to produce an increased amount of TSH. If the pituitary gland responds with increased production, then hypothalamic undersecretion may be the cause of the condition. If the pituitary does not increase secretion, it can indicate secondary hypothyroidism. Calcitonin is not secreted by the pituitary.

A patient is not able to absorb vitamin B12. The nurse determines that the patient is deficient in: a) Parietal (oxyntic) cells, which secrete HCl and intrinsic factor b) Peptic (chief cells), which secrete pepsinogen c) Mucous neck cells, which secrete mainly mucus d) S cells, which secrete secretin

a) Parietal (oxyntic) cells, which secrete HCl and intrinsic factor Parietal (oxyntic) cells secrete HCl and intrinsic factor, which is needed for vitamin B12 absorption. Mucous neck cells secrete mainly mucus; peptic (chief) cells, which secrete large quantities of pepsinogen, which is rapidly converted to pepsin. Secretin is secreted by the S cells and inhibits gastric acid secretion.

A nurse is caring for a patient who has been admitted to an inpatient facility for the treatment of anorexia nervosa. When planning the care of this patient, which of the following personality traits does the nurse anticipate observing? a) Perfectionistic and compulsive b) Introverted and shy c) Aggressive and extroverted d) Easily angered and paranoid

a) Perfectionistic and compulsive The causes of anorexia appear to be multifactorial, with determinants that include genetic influence; personality traits of perfectionism and compulsiveness; anxiety disorders; family history of depression and obesity; and peer, familial, and cultural pressures with respect to appearance. Being easily angered, paranoid, aggressive, extroverted, introverted, and shy are not necessarily associated with the typical personality traits of anorexia nervosa.

Which gland is often referred to as the master gland because it secretes many hormones? a) Pituitary b) Thyroid c) Hypothalamus d) Pancreas

a) Pituitary The pituitary gland has been called the master gland because its hormones control the functions of many target glands and cells. That is not a term used to refer to the other options.

After several months of persistent heartburn, a 57-year-old female client has been diagnosed with gastroesophageal reflux disease (GERD). Which treatment regimen is likely to best address the woman's health problem? a) Proton pump inhibitors; avoiding large meals; remaining upright after meals b) Antacids; avoiding positions that exacerbate reflux; a soft-textured diet c) Weight loss and administration of calcium channel blocking medications d) Surgical correction of the incompetent pylorus

a) Proton pump inhibitors; avoiding large meals; remaining upright after meals Proton pump inhibitors block the final stage of gastric acid production, effectively controlling the root cause of the esophageal damage associated with GERD. The pylorus is not involved, and a soft diet is not indicated. Calcium channel blocking drugs would not address the problem.

Select the most accurate statement regarding measurements of urinary hormone: a) Provide a better measure of hormone levels during a designated period. b) Requires blood and urine sampling. c) Discarded samples will not alter results. d) Drugs will not alter testing results.

a) Provide a better measure of hormone levels during a designated period. Measurements of urinary hormone or hormone metabolite excretion often are done on a 24-hour urine sample and provide a better measure of hormone levels during that period than hormones measured in an isolated blood sample. The advantages are relative ease of obtaining urine samples and blood sampling is not required. The disadvantages are that timed urine collections often are difficult to obtain and urine samples may be accidentally discarded or inaccurately preserved; drugs or disease states that alter hormone metabolism may interfere with the test results.

When acidic chyme enters the duodenum, it stimulates secretion of which of the following hormones? a) Secretin b) Gastrin c) Glucagon-like peptide-1 (GLP-1) d) Ghrelin

a) Secretin When stomach acid in chyme enters the intestine, it stimulates the release of secretin. Secretin inhibits gastrin secretion. Ghrelin is produced by endocrine cells in the fundus of the stomach. GLP-1 is produced in the distal small intestine, and its production is stimulated by a high-carbohydrate meal.

A client with malabsorption syndrome is experiencing fatty, bulky, yellow-gray, and foul-smelling stools. The nurse interprets this as: a) Steatorrhea b) Tetany c) Flatulence d) B12 deficiency

a) Steatorrhea A hallmark of malabsorption is steatorrhea, characterized by fatty, bulky, yellow-gray, and foul-smelling stools. The other options may occur as a result of the fat loss.

The results of a patient's 24-hour stool specimen indicate 20 g or more of fat. The nurse would interpret this as: a) Steatorrhea b) Diarrhea c) Hyperlipidemia d) Clostridium difficile

a) Steatorrhea Steatorrhea is the term used to describe fatty stools. It usually indicates that there is 20 g or more of fat in a 24-hour stool sample.

The cause of gastric carcinomas has been influenced by which of the following factors? Select all that apply. a) Strain of Helicobacter pylori (H. pylori) b) Family history c) Sexual history d) Younger age e) Environmental factors

a) Strain of Helicobacter pylori (H. pylori) b) Family history e) Environmental factors Chronic infection with H. pylori appears to serve as a cofactor in some types of gastric carcinomas. In addition to genetics, the likelihood of developing gastric cancer from an H. pylori infection is related to strain of H. pylori infection, environmental factors, and duration of infection.

Which of the following carbohydrates is matched to its correct enzyme needed for digestion? a) Sucrose and sucrase b) Starch and lactase c) Lactose and sucrose d) Starch and alpha dextrinase

a) Sucrose and sucrase Sucrose is a dietary carbohydrate digested by sucrase, an enzyme. This produces the monosaccharides fructose and glucose.

While educating a class about adipocytes in adipose tissue, the faculty will emphasize these cells not only serve as storage sites but also are able to: a) Synthesize triglycerides b) Produce linoleic fatty acid c) Increase glucagon release d) Degrade fat-soluble vitamins

a) Synthesize triglycerides Adipocytes synthesize and store lipids as triglycerides. Polyunsaturated linoleic fatty acid is obtained from dietary fats. Fat-soluble vitamins are components of the enzyme system required for energy to be released from fat. Adipocytes store glycerol and release it as an energy source when needed.

When a patient asks why some body fat is good, the nurse responds based on what fact? a) The body stores energy in adipose tissue b) Fat is required to repair the body c) The body does not need fat tissue d) Fat controls the amount of energy we have

a) The body stores energy in adipose tissue More than 90% of body energy is stored in adipose tissues of the body. The other options do not represent factual data about fat or adipose tissue.

A client is managing his diabetes with exercise and diet. The health care provider reviews the client's most recent lab results: fasting blood sugar level at 80 mg/dL and a hemoglobin A1C of 5%. Select the response that best identifies the client. a) The client is achieving normal glycemic control. b) The client is at risk for an insulin reaction. c) The client needs to modify his diet related to the low readings. d) The client is at risk for developing hyperglycemia.

a) The client is achieving normal glycemic control. The reading for the fasting blood sugar is appropriate, and the hemoglobin A1C level estimates good control of glucose levels over a 6- to 12-week period. This client should continue his exercise and diet routine as planned.

The nurse is providing education for a client diagnosed with hypothyroidism. What information about diet should the nurse be sure to include in this information? a) The client should maintain a low cholesterol diet. b) The client should increase the sodium content in her diet. c) Due to increased metabolism, the client will need to increase the caloric intake. d) The client should maintain a high protein diet.

a) The client should maintain a low cholesterol diet. Blood levels of cholesterol are decreased in hyperthyroidism and increased in hypothyroidism. The client may be placed on an antihyperlipidemic medication in addition to decreasing the cholesterol heavy foods in the diet.

A client has presented to a clinic for the treatment of severe dermatitis after contact with poison ivy on a camping trip. The client has been prescribed prednisone, a corticosteroid, for the treatment of his skin condition. The client's care provider has emphasized that dosages of the drug will be gradually tapered off rather than stopped upon resolution of the symptoms. Which rationale is the most accurate for this dosing protocol? a) The client's hypothalamic--pituitary--adrenal (HPA) system will require recovery time before normal function is restored. b) Abrupt cessation of the drug can contribute to symptoms similar to Cushing syndrome. c) Steroids can induce a dependency that is best addressed with a gradual withdrawal. d) HPA function is heightened during steroid administration and must return to normal levels before the drug is completely stopped.

a) The client's hypothalamic--pituitary--adrenal (HPA) system will require recovery time before normal function is restored. The suppression of the HPA system that accompanies steroid therapy requires time for a return to normal function. Dependency on the drug itself is not the rationale for tapering, and HPA function is suppressed, not heightened during therapy. Abrupt cessation can contribute to an Addison-like response, not Cushing syndrome

The nurse is teaching a client who has been newly diagnosed with hypothyroidism about the function of the thyroid. Which of these does the nurse explain to the client is the role of the thyroid gland? a) The thyroid gland is responsible for increasing the metabolic rate. b) The thyroid gland promotes development of secondary sex characteristics. c) The thyroid gland releases neurotransmitters when the "flight or flight" mechanism is stimulated. d) The thyroid gland is responsible for regulating serum calcium levels.

a) The thyroid gland is responsible for increasing the metabolic rate. The thyroid gland produces thyroid hormones, T3 and T4. These hormones increase the metabolic rate; increase protein and bone turnover; increase responsiveness to catecholamines; are necessary for fetal and infant growth and development. The parathyroid gland regulates calcium metabolism. The adrenal glands regulate flight or fight and the testes or ovaries regulate development of secondary sex characteristics.

The nurse is providing nutritional support education to the family of an obese 10-year-old. The most important information to provide would be: a) The weight loss intervention should include all family members. b) Reward the child's cooperation by increased television or computer time. c) Teach the child to skip the breakfast meal to decrease hunger through the day. d) Focus on having the child achieve ideal body weight.

a) The weight loss intervention should include all family members. The goal should be healthy eating and activity, not achievement of ideal body weight. The child's activities should be more active than watching television or using the computer. Skipping breakfast would not be recommended and should be eaten each day. The family should eat the same foods so as not to alienate the obese child.

Which of the following statements is true concerning gastric enterocytes? a) They secrete enzymes that aid in digestion of proteins. b) They provide binding sites for insulin. c) They secrete lubrication for the gastrointestinal tract. d) They digest entero bacteria.

a) They secrete enzymes that aid in digestion of proteins. Gastric enterocytes secrete enzymes that aid in the digestion of carbohydrates and proteins.

The nurse is teaching a new diabetic about fingerstick blood glucose testing. When the client obtains a pre-meal fingerstick reading of 206 mg/dL, which teaching by the nurse is most appropriate? a) This is an elevated reading - let's talk about adherence to insulin and diet. b) This is a normal reading for after meals. c) Blood glucose levels of 206 are acceptable when you are ill. d) This reading is within normal range - continue your regimen.

a) This is an elevated reading - let's talk about adherence to insulin and diet. Normal fasting or pre-meal blood glucose levels are between 70 and 110 mg/dL. The nurse should ensure the client understands the diet and medication regimen, as diabetes is a self-managed disease; maintaining good glucose control can prevent complications of the disease.

A client who has just undergone a thyroidectomy is experiencing high fever, tachycardia, and extreme restlessness. The nurse would interpret these manifestations as: a) Thyroid crisis b) Myxedematous coma c) Addisonian crisis d) Hypothyroidism

a) Thyroid crisis The symptoms this client is experiencing are related to thyroid crisis or storm and must be treated immediately to prevent death. Myxedematous coma is related to hypothyroidism but typically does not occur after a thyroidectomy. Addisonian crisis is related to hypoadrenalism.

The nurse is caring for a client who has obesity. The nurse recognizes that psychological factors may contribute to obesity. It would be most important for the nurse to assess the client for: a) Using food as a reward b) Time spent watching television c) Consumption of sugar-sweetened beverages d) Family eating patterns

a) Using food as a reward Psychological factors include using food as a reward, comfort, or means of getting attention. Eating may be a way to cope with tension, anxiety, and mental fatigue. The other choices are environmental influences.

Which of the following are functions of the digestive system? Select all that apply. a) Vitamins are synthesized. b) Nutrients are absorbed. c) Insulin is produced. d) Red blood cells are produced. e) Wastes are collected. f) Wastes are eliminated.

a) Vitamins are synthesized. b) Nutrients are absorbed. e) Wastes are collected. f) Wastes are eliminated. Nutrients are absorbed, wastes are collected and eliminated, and vitamins are synthesized in the digestive system.

While reviewing the colonic absorption and role of flora in the GI system, the instructor will stress that the large intestine contains: a) a complex microbial system that contains hundreds of different species. b) a few species of anaerobic bacteria and no aerobic bacteria. c) a high percentage of acidic secretions that limit bacterial invasion. d) only a few species of bacterial species, primarily aerobic in nature.

a) a complex microbial system that contains hundreds of different species. The stomach and small intestine contain only a few species of bacteria, probably because the composition of luminal contents (i.e., acids, bile, pancreatic secretions) kills most ingested microorganisms, and the propulsive movements of these organs impedes their colonization. The large intestine, on the other hand, contains a large and complex microbial ecosystem. It has been estimated that each individual has 300 to 500 different species of intestinal bacteria, with anaerobic bacteria outnumbering aerobic bacteria by a large percentage.

Release and synthesis of anterior pituitary hormones are mainly regulated by the inhibiting or releasing actions of the: a) hypothalamus. b) cell receptors. c) thymus gland. d) adrenal gland.

a) hypothalamus. The hypophysis (pituitary plus hypothalamus) and hypothalamus stimulatory hormones regulate the release and synthesis of anterior pituitary hormones. The adrenal gland and thymus gland hormones are regulated by the hypothalamus. Cell receptors are involved with the target cell response to the hormones.

A client is scheduled for a suppression test as part of the diagnostic testing for his suspected endocrine disorder. The results of this test will help the care team determine: a) whether the client is producing excessive hormone levels. b) if the client's hypothalamus is functioning correctly. c) if a target endocrine gland is capable of hormone production. d) whether the client's pituitary gland is functioning normally.

a) whether the client is producing excessive hormone levels. Suppression tests are used when hyperfunction of an endocrine organ is suspected. Suppression tests are not used to gauge pituitary or hypothalamus function. Stimulation tests are used to rule out hypofunction of an endocrine organ.

When educating a client with possible glucocorticoid dysfunction, the nurse will explain that the CRH controls the release of ACTH. The best time to perform the blood test to measure peak ACTH levels would be: a) 04:00 to 6:00 PM b) 06:00 to 08:00 AM c) 10:00 to 12:00 AM d) 09:00 to 11:00 PM

b) 06:00 to 08:00 AM Levels of cortisol increase as ACTH levels rise and decrease as ACTH levels fall. There is considerable diurnal variation in ACTH levels, which reach their peak in the early morning (around 6 to 8 AM) and decline as the day progresses.

The nurse is reviewing assessment data on four clients. Select the client at highest risk for developing type 2 diabetes. a) A 60-year-old female with a history of gestational diabetes b) A 45-year-old obese female with a sedentary lifestyle c) A 10-year-old male whose grandmother has type 2 diabetes d) A 40-year-old male who has an active lifestyle

b) A 45-year-old obese female with a sedentary lifestyle The person most at risk for developing type 2 diabetes is the 45-year-old obese female with a sedentary lifestyle. Other risk factors include family history, over age 40, and history of gestational diabetes. The 60-year-old would have developed it before age 60, if there were additional risk factors.

A client is diagnosed with hyperthyroidism and is exhibiting weight loss, diarrhea, and tachycardia. What does the nurse understand that these clinical manifestations are related to? a) A decrease in the level of glucose b) A hypermetabolic state c) A decrease in sympathetic nervous system activity d) A decrease in oxygen consumption

b) A hypermetabolic state Many of the manifestations of hyperthyroidism are related to the increase in oxygen consumption and use of metabolic fuels associated with the hypermetabolic state, as well as to the increase in sympathetic nervous system activity that occurs.

One of the earliest signs of Cushing syndrome is the loss of variable diurnal secretion of cortisol-releasing hormone (CRH) and which of the following? a) TSH b) ACTH c) DHEA d) GH

b) ACTH One of the earliest signs of Cushing syndrome, a disorder of cortisol excess, is the loss of diurnal variation in CRH and adrenocorticotropin hormone (ACTH) secretion, with corticotropin-releasing hormone (CRH) controlling the release of ACTH. The adrenal sex hormone dehydroepiandrosterone (DHEA) contributes to the pubertal growth of body hair, particularly pubic and axillary hair in women. Thyroid-stimulating hormone (TSH) levels are used to differentiate between primary and secondary thyroid disorders. Although secretion of growth hormone (GH) has diurnal variations over a 24-hour period, with nocturnal sleep bursts occurring 1 to 4 hours after onset of sleep, it is unrelated to ACTH and/or CRH secretion

A 79-year-old woman reports a recent onset of "nearly constant heartburn." During the assessment interview, she states that she has "lots of aches and pains." She states that she is not on any prescription medications but often takes aspirin for pain. The nurse should suspect what diagnosis? a) Staphylococcal infection b) Acute gastritis c) Gastric cancer d) Helicobacter pylori infection

b) Acute gastritis Aspirin is often implicated in cases of acute gastritis. H. pylori, gastric cancer, and staphylococcus infections do not normally cause reflux (heartburn) and are not directly linked to aspirin use.

Which enzyme is found in the mouth and is the first to initiate the breakdown of starches? a) Trypsin b) Amylase c) Bile d) Pepsin

b) Amylase Digestion of starch begins in the mouth with the action of amylase. Fat emulsification is influenced by bile. Protein digestion begins in the stomach with the action of pepsin and is further broken down by trypsin.

A client is to have a serum thyroxine and thyroid stimulating laboratory test performed to assess the baseline status of the hypothalamic-pituitary target cell hormones. When educating the client about the laboratory tests, when would the the nurse inform him the test should be obtained? a) After 0900 b) Before 0800 c) Prior to midnight d) At noon

b) Before 0800 The assessment of hypothalamic-pituitary function has been made possible by many newly developed imaging and radioimmunoassay methods. Assessment of the baseline status of the hypothalamic-pituitary target cell hormones involves measuring the following: ideally the laboratory specimens are obtained before 0800: serum cortisol, serum prolactin, serum thyroxine and TSH, serum testosterone and estrogen and serum LH/FSH, serum GH, and plasma and urine osmolality.

A child has been diagnosed with marasmus. Which clinical manifestations should the school nurse assess to confirm the diagnosis? Select all that apply. a) Pitting edema b) Bradycardia c) Stunted growth pattern d) Discolored hair e) Enlarged liver

b) Bradycardia c) Stunted growth pattern Inadequate food intake, with equal deficiencies of calories and protein, is the cause of marasmus, which is characterized by low heart rate, blood pressure, and body temperature; dull hair; and an emaciated appearance. Hair discoloration, enlarged liver, and pitting edema are manifestations of kwashiorkor, which is a severe protein deficiency.

Which complication of acromegaly can be life threatening? a) Bone overgrowth causes arthralgias b) Cardiac structures increase in size c) Splayed teeth result in impaired chewing d) Vertebral changes result in kyphosis

b) Cardiac structures increase in size While all the complications can exist, it is the enlargement of the heart and accelerated atherosclerosis that may lead to an early death. The teeth become splayed, causing a disturbed bite and difficulty in chewing. Vertebral changes often lead to kyphosis, or hunchback. Bone overgrowth often leads to arthralgias and degenerative arthritis of the spine, hips, and knees. Virtually every organ of the body is increased in size.

The newborn-nursery nurse is obtaining a blood sample to determine if a newborn has congenital hypothyroidism. What long-term complication is the nurse aware can occur if this test is not performed and the infant has congenital hypothyroidism? a) Irritability and restlessness b) Cretinism c) Dehydration from diarrhea d) Accelerated growth

b) Cretinism Congenital hypothyroidism is a common cause of preventable mental retardation. It affects approximately 1 in 4000 infants. The manifestations of untreated congenital hypothyroidism are referred to as cretinism. The term does not apply to the normally developing infant in whom replacement thyroid hormones therapy was instituted shortly after birth.

A nurse reading a sigmoidoscopy report notes that a patient was found to have skip lesions. The nurse interprets this as an indication of which of the following? a) Peptic ulcer b) Crohn's disease c) Zollinger-Ellison syndrome d) Ulcerative colitis

b) Crohn's disease Skip lesions, demarcated granulomatous lesions that are surrounded by normal-appearing mucosal tissue, are a characteristic feature of Crohn's disease.

Which effect of thyroid hormone deficit alters the function of all major organs in the body? a) Increases protein synthesis b) Decreases metabolism c) Causes vitamin deficiencies d) Enhances absorption of glucose

b) Decreases metabolism Thyroid hormone has two major functions: it increases metabolism and protein synthesis, and it is necessary for growth and development in children, including mental development and attainment of sexual maturity. Altered levels of thyroid hormone affect all the major organs in the body; hypothyroidism decreases metabolism and protein synthesis. Thyroid hormone deficit decreases the absorption of glucose from the gastrointestinal tract. Because vitamins are essential parts of metabolic enzymes and coenzymes, an increase (rather than decrease) in metabolic rate causes the use of vitamins and tends to cause vitamin deficiency

When caring for a client with anemia and a decrease in red blood cells (RBCs), the nurse recognizes which of these hormones will stimulate the bone marrow to produce additional RBCs? a) Somatostatin b) Erythropoietin c) Cortisol d) Atrial natriuretic peptide

b) Erythropoietin Erythropoietin is made in the kidney and stimulates erythropoiesis, production of RBCs in the bone marrow

A 23-year-old man is admitted to the hospital. He is experiencing polyphagia, polyuria, and polydipsia. He states that the condition has come on very suddenly. This client is likely to require what treatment? a) Biguanides b) Exogenous insulin injections c) Sulfonylureas

b) Exogenous insulin injections Type 1 diabetes is the best diagnosis as the client has no other symptoms and the disease presented so rapidly. Insulin will likely be necessary and oral antihyperglycemics will not be sufficient.

A nurse on a medical unit is providing care for a 37-year-old female client who has a diagnosis of Graves' disease. Which assessments should the nurse prioritize? a) Cognition and judgment b) Eye health and visual acuity c) Skin integrity and distribution of adipose tissue d) Signs and symptoms of decreased bone density

b) Eye health and visual acuity The ophthalmopathy of Graves' disease can cause severe eye problems, including tethering of the extraocular muscles resulting in diplopia; involvement of the optic nerve, with some visual loss; and corneal ulceration because the lids do not close over the protruding eyeball (due to the exophthalmos). Eye assessment is consequently a priority over assessment of skin integrity, cognition, or musculoskeletal status.

The swallowing reflex is an entirely voluntary activity. a) True b) False

b) False Although swallowing is initiated as a voluntary activity, it becomes involuntary as food or fluid reaches the pharynx.

While discussing the regulation of hormone levels, the instructor gives an example of hormones regulated by feedback mechanisms. Which example of this regulation is best? a) When a female is thinking about getting pregnant, the body knows to release female sex hormones in greater proportion than usual. b) Following a meal that was high in carbohydrates, a person's blood glucose elevates, which stimulates the release of insulin from the pancreas. c) In children, the body knows to release growth hormones while they are sleeping instead of when they are awake and running around. d) When a person's body pH is decreasing, the stomach lining becomes more acidic to offset this.

b) Following a meal that was high in carbohydrates, a person's blood glucose elevates, which stimulates the release of insulin from the pancreas. The levels of hormones such as insulin and antidiuretic hormone (ADH) are regulated by feedback mechanisms that monitor substances such as glucose (insulin) and water (ADH) in the body. None of the other examples are accurate examples of this feedback mechanism.

Advances in technology have made it possible to assess hypothalamic-pituitary function by newly developed imaging and radioimmunoassay methods. When baseline tests are not sufficient, what suppression test gives information about combined hypothalamic-pituitary function? a) ACTH suppression test b) GH suppression test c) Cortisol suppression test d) Prolactin suppression test

b) GH suppression test When further information regarding pituitary function is required, combined hypothalamic-pituitary function tests are undertaken (although these are performed less often today). These tests consist mainly of hormone stimulation tests (e.g., rapid ACTH stimulation test) or suppression tests (e.g., GH suppression test). The other answers are incorrect.

A patient with diabetes asks the nurse for advice in controlling between-meal blood glucose levels. Which of the following might the nurse suggest as a long-acting insulin to provide a consistent basal level? a) Glulisine (Apidra) b) Glargine (Lantus) c) Regular d) Aspart (Novolog)

b) Glargine (Lantus) Glargine is a long-acting peakless insulin that provides consistent basal levels over a 20- to 24-hour period. Regular insulin has a duration of 5 to 7 hours . Aspart and glulisine have the most rapid onset and shortest duration of action, making them optimal for postprandial glucose control.

A client with Graves' disease has had radioiodine treatment with worsening of ophthalmopathy. What medication does the nurse prepare to administer that the client will use for several weeks to decrease these symptoms? a) Diphenhydramine (Benadryl) b) Glucocorticoids c) Beta-adrenergic blockers d) Levothyroxine (Synthroid)

b) Glucocorticoids Some physicians prescribe glucocorticoids for several weeks surrounding the radioiodine treatment if the person had signs of ophthalhmopathy.

A client tells the health care provider that he has been very compliant over the last 2 months in the management of his diabetes .The best diagnostic indicator that would support the client's response would be: a) Urine test b) Glycosylated hemoglobin, hemoglobin A1C (HbA1C) c) Fasting blood glucose level d) Capillary blood glucose sample

b) Glycosylated hemoglobin, hemoglobin A1C (HbA1C) Glycosylated hemoglobin, hemoglobin A1C (HbA1C), and A1C are terms used to describe hemoglobin into which glucose has been incorporated. Glycosylation is essentially irreversible, and the level of A1C present in the blood provides an index of blood glucose levels over the previous 6 to 12 weeks. In uncontrolled diabetes or diabetes with hyperglycemia, there is an increase in the level of A1C. The other options would not reflect the 2-month period.

Select the statement that best explains the function of hormone receptors. a) Hormone receptors produce antibodies to block certain hormones from entering the cell. b) Hormone receptors recognize a specific hormone and translate the signal into a cellular response. c) Hormone receptors respond to decreasing hormone levels by producing a decrease in receptor numbers. d) Hormone receptors are located within the red blood cells and carry hormones to the target cells.

b) Hormone receptors recognize a specific hormone and translate the signal into a cellular response. Hormone receptors are complex molecular structures that are located either on the surface of or inside target cells. The function of these receptors is to recognize a specific hormone and translate the hormonal signal into a cellular response. The other choices are not true

While assessing a client who is experiencing diarrhea caused by Clostridium difficile, the nurse should anticipate hearing: a) Normal bowel sounds b) Hyperactive bowel sounds c) Hypoactive bowel sounds d) Absence of bowel sounds

b) Hyperactive bowel sounds Inflammatory changes increase motility, which would lead to hyperactive bowel sounds. In many instances, it is not certain whether changes in motility occur because of inflammation or are secondary to the effects of toxins or unabsorbed materials.

A man is brought into the emergency department by paramedics who state that the client passed out on the street. The man smells of alcohol, and when roused says he has not eaten since yesterday. He is wearing a medic alert bracelet that says he is a diabetic. What would the nurse suspect as a diagnosis? a) Hypernatremia b) Hypoglycemia c) Hyperglycemia d) Hyponatremia

b) Hypoglycemia Alcohol decreases liver gluconeogenesis, and people with diabetes need to be cautioned about its potential for causing hypoglycemia, especially if alcohol is consumed in large amounts or on an empty stomach.

The physician is assessing a client with a preliminary diagnosis of endocrine disorder. Further assessment findings identify abnormalities with emotion, pain, body temperature, and neural input. The physician determines the need to further assess the: a) Cerebellum b) Hypothalamus c) Anterior pituitary d) Cerebral cortex

b) Hypothalamus The hypothalamus is the coordinating center of the brain for endocrine, behavioral, and autonomic nervous system function. It is at the level of the hypothalamus that emotion, pain, body temperature, and other neural input are communicated to the endocrine system. The anterior pituitary regulates several physiological processes, including stress, growth, reproduction, and lactation. The cerebellum is involved in motor control, and the cerebral cortex is associated with sensory, motor, and association

Colonic microorganisms play a role in the synthesis of which of the following vitamins? a) C b) K c) D d) E

b) K Colonic microorganisms play a role in vitamin synthesis and in absorption of calcium, magnesium, and iron. Colonic flora synthesize vitamin K.

A patient informs the nurse that he or she feels as though he or she have a kidney stone again. The patient is on a diet for weight loss and feels as though this may be a contributing factor as it never occurred before going on the diet. Which of the following diets does the nurse suspect the patient may be on? a) 1200-calorie American Diabetic Association diet b) Low-carbohydrate/high protein c) Low-fat, low-protein, high-carbohydrate d) Vegetarian

b) Low-carbohydrate/high protein Carbohydrate diets are effective for weight loss, especially in the initial stages but can contribute to health risks. Higher protein diets can increase the risk of kidney stones, and the decrease in fiber can also increase risks of cancer and raise cholesterol levels.

A diabetic client was visiting the endocrinologist for annual checkup. The client's blood work reveals an increased level of which lab result that reveals early signs of diabetic nephropathy? a) Oliguria b) Microalbuminuria c) Hyperlipidemia d) Hypokalemia

b) Microalbuminuria One of the first manifestations of diabetic nephropathy is increased urinary albumin excretion (i.e., microalbuminuria). Risk factors, rather than renal manifestations, include glycosylated hemoglobin levels greater than 8.1%, genetic and familial predisposition, hypertension, poor glycemic control, smoking, and hyperlipidemia. Usually, serum potassium levels are elevated (hyperkalemia) in diabetic nephropathy. The presence of ketones in the urine is a sign of ketoacidosis and severe hyperglycemia rather than nephropathy

When explaining to a group of parents of school-aged children the importance of protein intake, the nurse emphasizes which food choices that contain all essential amino acids in adequate amounts per serving? Select all that apply. a) French fries b) Milk c) Frozen pizza bites d) Chicken e) Eggs

b) Milk d) Chicken e) Eggs Proteins are composed of amino acids, nine of which leucine, isoleucine, methionine, phenylalanine, threonine, tryptophan, valine, lysine, and histidine. The foods that provide these essential amino acids in adequate amounts are milk, eggs, meat, fish, and poultry. Obviously, french fries and pizza bites are not great sources of protein.

Parents are concerned that their child may be at risk for the development of childhood obesity. Select the factors that would place a child at risk. Select all that apply. a) Regular balanced eating pattern b) Mother had gestational diabetes during pregnancy c) Excessive exercise d) Ethnicity e) Heredity f) Underweight parents

b) Mother had gestational diabetes during pregnancy d) Ethnicity e) Heredity Childhood obesity is determined by a combination of hereditary and environmental factors. It is associated with obese parents, gestational diabetes and excessive weight gain during pregnancy, formula feeding, parenting style, parental eating habits, energy-dense food choices, erratic eating patterns, ethnicity, and a sedentary lifestyle.

The incidence of stomach cancer has significantly decreased in the United States, yet it remains the leading cause of death worldwide. The nurse understands the reason for the high mortality rate in stomach cancer is because of which of the following reasons? a) Diagnostic testing is not available for detection of the disease. b) Patients have few early symptoms of the disease. c) Patients are afraid to talk about their symptoms. d) The progression of the disease is rapid.

b) Patients have few early symptoms of the disease. Although the incidence of cancer of the stomach has declined over the past 50 years in the United States, it remains the leading cause of death worldwide. Because there are few early symptoms with this form of cancer, the disease is often far advanced at the time of diagnosis. Diagnosis of gastric cancer is accomplished by a variety of techniques, including barium x-ray studies, endoscopic studies with biopsy, and cytologic studies (e.g., Papanicolaou smear) of gastric secretions. Chronic infection with Helicobacter pylori appears to serve as a cofactor in some types of gastric carcinomas. The bacterial infection causes gastritis, followed by atrophy, intestinal metaplasia, and carcinoma.

Upon admission, a patient tells the nurse that he takes aspirin every 4 hours every day. The nurse determines that this patient is at risk for which of the following? a) Cancer of the stomach b) Peptic ulcer c) Zollinger-Ellison syndrome d) Crohn's disease

b) Peptic ulcer Peptic ulcers occur in the areas of the upper gastrointestinal tract and are caused by Helicobacter pylori infection and aspirin or nonsteroidal anti-inflammatory drug use. Therefore, the nurse determines that a patient taking aspirin every 4 hours daily is at risk for a peptic ulcer.

Select the most common symptoms of diabetes. Select all that apply. a) Polyhydramnios b) Polydipsia c) Polyuria d) Polycythemia e) Polyphagia

b) Polydipsia c) Polyuria e) Polyphagia The most commonly identified signs and symptoms of diabetes are often referred to as the three polys: (1) polyuria (i.e., excessive urination), (2) polydipsia (i.e., excessive thirst), and (3) polyphagia (i.e., excessive hunger). Polyhydramnios is a medical condition describing an excess of amniotic fluid in the amniotic sac. Polycythemia is a condition of increased red blood cells.

As part of maintaining homeostasis, hormones secreted by endocrine cells are inactivated continuously to: a) Stimulate production b) Prevent accumulation c) Absorb metabolic waste d) Free receptor sites

b) Prevent accumulation Continuous inactivation of secreted hormones is necessary to prevent accumulation that could disrupt the feedback mechanism. Increased secretion stimulates production of more receptor sites. Metabolic waste absorption is not a function of the endocrine system.

Which of the following statements is true concerning food digestion? a) Monosaccharides cannot be absorbed. b) Protein digestion begins in the stomach. c) Pepsin helps to digest food in the small intestine. d) Proteins are broken down by carbohydrates.

b) Protein digestion begins in the stomach. Protein digestion begins in the stomach with the action of pepsin. Pepsin cannot act in the small intestine, as it is broken down by the alkaline pH in the gastro intestinal tract. Proteins do not break down carbohydrates, and monosaccharides can be absorbed.

The nurse is providing dietary instruction to a client whose lab values indicate a high level of blood cholesterol. The client asks if there are any food contents that need to be avoided. The best response would be: a) Double-bond fatty acids b) Saturated fatty acids c) Nonsaturated fatty acids d) Polyunsaturated fatty acids

b) Saturated fatty acids The saturated fatty acids elevate blood cholesterol, whereas the monounsaturated and polyunsaturated fats lower blood cholesterol.

A client in a nursing home complains to her nurse that she is not feeling well. When asked to describe how she feels, the client states that she really is not hungry anymore and seems to have indigestion a lot. The nurse checks the client's chart and finds that her vital signs are normal, but that she has lost weight over the past 2 months. She also notes that there is a history of gastric cancer in the client's family. The nurse notifies the physician and expects to receive what orders? (Select all that apply.) a) Schedule a lower gastrointestinal study b) Schedule a barium radiograph and an endoscopy c) Order cytologic studies to be done during the endoscopy d) Perform a Papanicolaou smear on the client's gastric secretions e) Have the technician do an endoscopic ultrasound

b) Schedule a barium radiograph and an endoscopy c) Order cytologic studies to be done during the endoscopy Diagnosis of gastric cancer is accomplished by means of a variety of techniques, including barium radiographic studies, endoscopic studies with biopsy, and cytologic studies (e.g., Papanicolaou smear) of gastric secretions. Cytologic studies can prove particularly useful as routine screening tests for persons with atrophic gastritis or gastric polyps. Computed tomography and endoscopic ultrasonography often are used to delineate the spread of a diagnosed stomach cancer. Papanicolaou smears are done on gastric secretions but not by the nurse. A lower gastrointestinal study would be of no value in diagnosing this client. A technician does not do an endoscopic ultrasound

A client with diabetes carries insulin with him at all times. At 11:35, he obtains a blood glucose reading of 12.1 mmol/l and self-administers a dose of insulin in anticipation of eating lunch at noon. What type of insulin did he most likely inject? a) Premixed b) Short acting c) Intermediate acting d) Long acting

b) Short acting Short acting (regular) insulin is used to facilitate metabolism of the food that is being eaten. Premixed insulin is possible, but is not used as commonly. Intermediate- and long-acting insulin would have too distant an onset.

A parent arrives in the endocrinology clinic with her 8-year-old son, concerned about his rapid development and tall stature. What significant assessment finding does the nurse recognize is important to report to the physician related to the development of precocious puberty? a) Multiple dental caries b) Significant genital enlargement c) Enlarged head circumference d) The child is 20 pounds over his target weight for height

b) Significant genital enlargement Diagnosis of precocious puberty is based on physical findings of early thelarche, adrenarche, and menarche. The most common sign in boys is early genital enlargement. Radiologic findings may indicate advanced bone age. People with precocious puberty are unusually tall for their age as children but short as adults because of the early closure of the epiphyses.

Research has identified a cycle of insulin-induced posthypoglycemic episodes. What is this phenomenon called? a) Joslin phenomenon b) Somogyi effect c) Sunset effect d) Dawn phenomenon

b) Somogyi effect The Somogyi effect describes a cycle of insulin-induced posthypoglycemic episodes. In 1924, Joslin and associates noticed that hypoglycemia was associated with alternate episodes of hyperglycemia. The other answers are not correct.

A nurse administering a patient's medication tells the patient that a proton pump inhibitor has been added. When the patient asks the purpose of the medication, the nurse responds that it is to prevent which of the following? a) Crohn's disease b) Stress ulcer c) Malabsorption syndrome d) Ulcerative colitis

b) Stress ulcer Proton pump inhibitors are the first line of medications used in the prevention of stress ulcers.

The nurse is providing discharge instructions for a client with Graves' disease who has ophthalmopathy. What should the nurse be sure to include in the instructions to decrease exacerbation of this clinical manifestation? a) The client should be informed that he should not be in contact with other people during the acute phase. b) The client should be strongly urged not to smoke. c) The client should be informed that if he begins to feel symptoms getting worse, he should take an extra dose of their medication. d) The client should be strongly encouraged not to drink any alcohol.

b) The client should be strongly urged not to smoke. Ophthalmopathy can also be aggravated by smoking, which should be strongly discouraged. It is not necessary for the client to avoid contact with others. Alcohol is not contraindicated but should be limited when taking any medication regimen. The client should not adjust the doses of medications without first consulting the physician.

A patient is managing his type 2 diabetes with exercise and diet. He has a fasting blood sugar level (FBS) of 80 mg/dL and a hemoglobin A1C of 5%. Based on these findings, which of the following can the nurse assume? a) The patient is at risk for an insulin reaction. b) The patient is achieving normal glycemic control. c) The patient needs to modify his diet related to the low readings. d) The patient is at risk for developing hyperglycemia.

b) The patient is achieving normal glycemic control. The reading for the FBS is appropriate (<100 mg/dL is normal) and the hemoglobin A1C level (<6.5) shows good control of glucose levels over a 6- to 12-week period. This patient should continue his current exercise and diet routine, which is working well for him

A nurse assessing a female patient obtains a waist circumference of 92 cm from the patient. Which of the following does this finding indicate to the nurse? a) This finding is appropriate for a female patient. b) The patient may have increased health risks associated with this finding. c) The patient is at risk for increased psychological disorders with this finding. d) The patient will be able to have a quicker weight loss.

b) The patient may have increased health risks associated with this finding. A waist circumference of 88 cm or greater in women and 102 cm or greater in men has been associated with increased health risk.

Which of the following statements most accurately describes the function of the secretory glands in the gastrointestinal (GI) tract? a) Each day approximately 2000 mL of fluid is secreted into the GI tract. b) The secretory glands produce mucus to lubricate and protect the mucosal layer of the GI tract wall. c) The secretions are mainly albumin with sodium and potassium. d) Secretory activity is increased with sympathetic stimulation.

b) The secretory glands produce mucus to lubricate and protect the mucosal layer of the GI tract wall. Secretory glands in the GI tract serve two basic functions: production of mucus to lubricate and protect the GI tract wall and secretion of fluids and enzymes to aid in digestion and absorption of nutrients. Each day approximately 7000 mL of fluid is secreted into the GI tract. Secretory activity is increased with parasympathetic stimulation and inhibited with sympathetic activity. The secretions are mainly water with sodium and potassium.

A client with long-standing type 2 diabetes is surprised at his high blood sugar readings while recovering from an emergency surgery. Which of the following factors may have contributed to the client's inordinately elevated blood glucose levels? a) Illness inhibited the release and uptake of glucagon. b) The stress of the event caused the release of cortisol. c) The tissue trauma of surgery resulted in gluconeogenesis. d) Sleep disruption in the hospital precipitated the dawn effect.

b) The stress of the event caused the release of cortisol. Elevation of glucocorticoid levels, such as during stressful events, can lead to hyperglycemia. Tissue trauma does not cause gluconeogenesis, and illness does not inhibit the action of glucagon. The dawn phenomenon is not a likely cause of the client's disruption in blood sugar levels.

Malabsorption syndrome results in loss of fat in the stool and failure to absorb fat-soluble vitamins. a) False b) True

b) True In malabsorption syndrome, there is loss of fat in the stools and failure to absorb the fat-soluable vitamins.

The health care provider reviews the waist-hip ratio of a male. The ratio is 1.0. The provider would interpret his result as: a) Peripheral obesity b) Upper body obesity c) Lower body obesity d) Gluteal-femoral obesity

b) Upper body obesity A waist-hip ratio greater than 1.0 in men and 0.8 in women indicates upper body obesity. Research suggests that fat distribution may be a more important factor for morbidity and mortality than overweight or obesity. The other options are lower body obesity types.

A public health nurse is teaching a group of young adults about actions to prevent obesity and promote general health. The nurse should encourage: a) exercising for at least 1 hour, five times per week. b) doing physical activity for at 30 minutes daily. c) eliminating dietary fats whenever possible. d) the consumption of organic foods whenever possible.

b) doing physical activity for at 30 minutes daily. Current recommendations include 30 minutes of physical exercise daily. Organic foods tend to be healthy, but this is not a central focus of obesity prevention. Excess fat intake must be avoided, but fats are a necessary dietary component that must not be eliminated.

A 66-year-old woman has been diagnosed with diverticular disease based on her recent complaints and the results of a computed tomography (CT) scan. Which of the client's following statements demonstrates an accurate understanding of this diagnosis? a) "I think this might have happened because I've used enemas and laxatives too much." b) "I've always struggled with heartburn and indigestion, and I guess I shouldn't have ignored those warning signs." c) "I suppose I should try to eat a bit more fiber in my diet." d) "From now on, I'm going to stick to an organic diet and start taking more supplements."

c) "I suppose I should try to eat a bit more fiber in my diet." Increased fiber is important in both the prevention and treatment of diverticular disease. Overuse of laxatives is not linked to diverticular disease, and heartburn and indigestion are not specific signs of the problem. An organic diet and the use of dietary supplements are not key treatments.

A client with a new diagnosis of type 2 diabetes mellitus states, "I am really worried that I might need to take injections. Is there something I can do to avoid that?" What is the best response by the nurse? a) "If you exercise weekly, you won't need injections." b) "Diabetes management is complicated; you can't avoid injections." c) "You could regulate your diet, exercise regularly, and lose weight." d) "A support group could help you cope with stress and learn helpful tips."

c) "You could regulate your diet, exercise regularly, and lose weight." Diabetes management is based on dietary regulation, exercise, and medications. Many clients with type 2 diabetes mellitus can avoid injections if they exercise regularly, follow dietary guidelines, and manage their weight

Which statement is true concerning energy requirements across the lifespan? a) A pregnant woman needs an additional 500 kcal/ day. b) Regardless of gender, adolescents require 50 kcal/kg of body weight. c) A newborn requires more kcal/kg of body weight than a 10 year old. d) Growth spurts require additional protein but no additional kcal.

c) A newborn requires more kcal/kg of body weight than a 10 year old. Energy requirements are greater during growth periods. A person requires approximately 115 kcal/kg of body weight at birth, 105 kcal/kg at 1 year of age, and 80 kcal/kg from 1 to 10 years of age. During adolescence, boys require 45 kcal/kg of body weight and girls require 38 kcal/kg. During pregnancy, a woman needs an extra 300 kcal/day above her usual requirement, and during the first 3 months of breast-feeding she requires an additional 500 kcal

A lung cancer client with small cell carcinoma may secrete an excess of which hormone causing an ectopic form of Cushing syndrome due to a nonpituitary tumor? a) DHEA b) TSH c) ACTH d) GH

c) ACTH The third form (of Cushing syndrome) is ectopic Cushing syndrome, caused by a nonpituitary ACTH-secreting tumor. Certain extra pituitary malignant tumors such as small cell carcinoma of the lung may secrete ACTH or, rarely, CRH and produce Cushing syndrome. The adrenal sex hormone dehydroepiandrosterone (DHEA) contributes to the pubertal growth of body hair, particularly pubic and axillary hair in women. Thyroid-stimulating hormone (TSH) levels are used to differentiate between primary and secondary thyroid disorders. Although secretion of growth hormone (GH) has diurnal variations over a 24-hour period, with nocturnal sleep bursts occurring 1 to 4 hours after onset of sleep, it is unrelated to ACTH and/or CRH secretion.

The nurse is caring for a 42-year-old male client who is admitted for treatment of heart failure. He has abnormally large hands and feet and a broad face with a protruding jaw. Based on these signs and symptoms, the nurse identifies which of the following endocrine disturbances as the most likely cause for these physical changes? a) Cushing syndrome b) Hyperthyroidism c) Acromegaly d) Myxedema

c) Acromegaly Enlargement of the small bones of the hands and feet and of the membranous bones of the face and skull results in a pronounced enlargement of the hands and feet, a broad and bulbous nose, a protruding jaw, and a slanting forehead. Bone overgrowth often leads to arthralgias and degenerative arthritis of the spine, hips, and knees. Virtually every organ of the body is increased in size. Enlargement of the heart and accelerated atherosclerosis may lead to an early death. Hyperthyroidism results from excess thyroid hormone. Myxedema and Cushing syndrome are the result of adrenal abnormalities and do not cause these bone changes.

The nurse is assessing a client with thyrotoxicosis and the nurse is explaining how the thyroid gland is stimulated to release thyroid hormones. The nurse should describe what process? a) Steady-state continuous release b) Homeostatic receptors on surface of gland c) Action of releasing hormones from hypothalamus d) Direct neural stimulation

c) Action of releasing hormones from hypothalamus The synthesis and release of anterior pituitary hormones are largely regulated by the action of releasing or inhibiting hormones from the hypothalamus, which is the coordinating centre of the brain for endocrine activity such as thyroid activity. There are no direct innervations for hormone release from the thyroid gland, and homeostatic receptors do not exist. Steady-state release of hormones does not occur.

Hormones are usually divided into categories according to their structure. The release of epinephrine would be classified as: a) Peptides and polypeptides b) Steroids c) Amines and amino acids d) Proteins

c) Amines and amino acids Hormones are divided into three categories according to their structures: amines and amino acids; polypeptides, proteins, and glycoproteins; and steroids. The amine and amino acid hormones include norepinephrine and epinephrine, which are derived from a single amino acid (i.e., tyrosine). The peptide, polypeptide, protein, and glycoprotein hormones can be as small as thyrotropin-releasing hormone (TRH), which contains three amino acids, and as large, and as large and complex as growth hormone (GH) and follicle-stimulating hormone (FSH). Steroid hormones, such as the glucocorticoids, are derivatives of cholesterol.

Which of the following functions is performed by saliva? a) Catalysis of brush border enzymes b) Buffering of gastric secretions c) Antimicrobial protection d) Promotion of intestinal flora

c) Antimicrobial protection Saliva performs three roles: lubrication, antimicrobial protection, and initiation of starch digestion. Saliva does not promote the growth of intestinal flora, buffer gastric secretions, or catalyze the actions of brush border enzymes.

The pancreas is an endocrine organ that is composed of the acini and the islets of Langerhans. The islets of Langerhans have alpha, beta, and delta cells as well as the PP cell. Which cells secrete insulin? a) Alpha cells b) Delta cells c) Beta cells d) PP cells

c) Beta cells Each islet is composed of beta cells that secrete insulin and amylin, alpha cells that secrete glucagon, and delta cells that secrete somatostatin. In addition, at least one other type of cell, the PP cell, is present in small numbers in the islets and secrets a hormone of uncertain function called pancreatic polypeptide

The nurse is planning to collect a 24-hour urine sample for hormone assay. In which of these situations does the nurse collaborate with the health care provider to find an alternate type of testing? a) Client has anemia. b) Client has diabetes. c) Client has anuria. d) Client has hypothyroidism.

c) Client has anuria. The advantages of a urine test include the relative ease of obtaining urine samples and the fact that blood sampling is not required. The disadvantage is that reliably timed urine collections often are difficult to obtain and rely on adequate renal function. Anuria refers to the absence of urine output.

A nurse is speaking at a weight loss meeting about the dangers to one's health from obesity. Which condition is directly attributed to being obese? a) Dysrhythmias b) Recurrent infections c) Coronary heart disease d) Increased bone resorption

c) Coronary heart disease Overweight and obesity have become global health problems and are associated with an increased risk health problems that include cardiac disease. Bone resorption is not directly affected and obese individuals are not noted to have recurrent infections. Dysrhythmias are not an identified effect of obesity.

A patient with type 2 diabetes experiences unexplained elevations of fasting blood glucose in the early morning hours. Which of the following conditions can account for this effect? a) Autonomic neuropathy b) Diabetic ketoacidosis c) Dawn phenomenon d) Inadequate bedtime insulin

c) Dawn phenomenon The dawn phenomenon involves increased levels of fasting blood glucose or insulin requirement between the hours of 5 and 9 in the morning. It is not preceded by hypoglycemia. Circadian release of growth hormone and cortisol may be contributing factors. The other answer selections are not characterized by increased early morning levels of blood glucose.

A patient experiences an increase in thyroid hormone as a result of a thyroid tumor. Which of the following hormonal responses demonstrates the negative feedback mechanism? a) Decreased adrenocorticotropic hormone b) Increased follicle-stimulating hormone c) Decreased thyroid-stimulating hormone (TSH) d) Increased thyrotropin-releasing hormone

c) Decreased thyroid-stimulating hormone (TSH) Negative feedback occurs when secretion of one hormone causes a reduction in the secretion of the hormone that stimulates production of the first hormone. In this case, TSH, which is manufactured by the anterior pituitary gland, would normally stimulate release of thyroid hormones, but with the increase of those hormones by the secreting tumor, enough thyroid hormones flood the system that there should be a reduction in TSH levels.

The physiological rationale for hanging normal saline (0.9% NS) or 5% dextrose in water (D5W) to a client who has been experiencing diarrhea includes: a) Emulsification of fats b) Activation of the ATP channels c) Facilitating the absorption of osmotically active particles d) Activating the pancreatic enzymes of trypsin and elastase

c) Facilitating the absorption of osmotically active particles Water absorption from the intestine is linked to absorption of osmotically active particles, such as glucose and sodium. It follows that an important consideration in facilitating the transport of water across the intestine (and decreasing diarrhea) after temporary disruption in bowel function is to include sodium and glucose in the fluids that are consumed. None of the other distractors addresses this principle. Activating the pancreatic enzymes of trypsin and elastase is needed for protein digestion and absorption. Emulsification of fats begins in the stomach and continues in the duodenum under the influence of bile from the liver.

A teenager who has a history of achalasia will likely complain of which of the following clinical manifestations? a) Excessive heartburn following a high-fat meal of French fries b) Projectile vomiting across the room unrelated to meals c) Feeling like there is food stuck in the back of the throat d) Vomiting large amounts of bright red emesis

c) Feeling like there is food stuck in the back of the throat Achalasia produces functional obstruction of the esophagus so that food has difficulty passing into the stomach, and the esophagus above the lower esophageal sphincter becomes distended. Symptoms following high-fat intake is usually associated with gallbladder disease. Projectile vomiting is usually related to increased intracranial pressure. Vomiting blood can be associated with esophagitis, erosion of the esophagus, bleeding esophageal varices, or esophageal cancer.

A 25-year-old female client exhibits exophthalmos of both eyes. The health care provider recognizes this as a manifestation of: a) Hashimoto thyroiditis b) Myxedema c) Graves disease d) Acquired hypothyroidism

c) Graves disease Graves disease is a state of hyperthyroidism in which opthalmopathies, such as exophthalmos, typically occur. The other conditions are states of hypothyroidism and are not associated with this abnormality

The nurse is performing an assessment for a client who has hyperthyroidism that is untreated. When obtaining vital signs, what is the expected finding? a) Temperature 96oF b) Respiratory rate 14 c) Heart rate 110 and bounding d) Blood pressure 180/110 mm Hg

c) Heart rate 110 and bounding Cardiovascular and respiratory functions are strongly affected by thyroid function. With an increase in metabolism, there is a rise in oxygen consumption and production of metabolic end products, with an accompanying increase in vasodilation. Blood volume, cardiac output, and ventilation are all increased. Heart rate and cardiac contractility are enhanced as a means of maintaining the needed cardiac output. Blood pressure is likely to change little because the increase in vasodilation tends to offset the increase in cardiac output.

The nurse is teaching a client diagnosed with Addison disease about the importance of lifetime oral replacement therapy. Select the pharmacologic agent that would be prescribed. a) Ketoconazole b) Potassium supplements c) Hydrocortisone d) Insulin

c) Hydrocortisone Hydrocortisone is usually the drug of choice in treating Addison disease. In mild cases, hydrocortisone alone may be adequate. Ketoconazole causes excessive breakdown of glucocorticoids and can also result in adrenal insufficiency. Clients with Addison disease usually have elevated potassium levels, and insulin is not the treatment for Addison disease.

Which gland acts as a signal relaying bridge between multiple body systems and the pituitary gland? a) Thyroid b) Posterior pituitary c) Hypothalamus d) Parathyroid

c) Hypothalamus The activity of the hypothalamus is regulated by both hormonally mediated signals (e.g., negative feedback signals) and by neuronal input from a number of sources. Neuronal signals are mediated by neurotransmitters such as acetylcholine, dopamine, norepinephrine, serotonin, ?-aminobutyric acid (GABA), and opioids. Cytokines that are involved in immune and inflammatory responses, such as the interleukins, also are involved in the regulation of hypothalamic function. This is particularly true of the hormones involved in the hypothalamic-pituitary-adrenal axis. Thus, the hypothalamus can be viewed as a bridge by which signals from multiple systems are relayed to the pituitary gland. This cannot be said of the other options.

Crohn's disease is treated by several measures. Treatment with corticosteroids focuses on which of the following? a) Decreased bleeding tendency b) Immune suppression c) Inflammatory suppression d) Increased appetite

c) Inflammatory suppression Treatment methods focus on terminating the inflammatory response and promoting healing, maintaining adequate nutrition, and preventing and treating complications. Several medications have been successful in suppressing the inflammatory reaction, including corticosteroids, sulfasalazine, metronidazole, azathioprine, 6-mercaptopurine, methotrexate, and infliximab.

Crohn's disease not only affects adults but also can occur in children. The nurse assesses for which of the following major manifestations in children with Crohn's disease? a) Halitosis b) Dental caries c) Malnutrition d) Weight gain

c) Malnutrition When Crohn's disease occurs in children, one of the major manifestations may be retardation of growth and significant malnutrition.

The nurse is providing education to a client who has been instructed to increase the amount of protein in her diet. Which foods should the nurse recommend? a) Nuts and seeds b) Whole-wheat bread and green vegetables c) Milk and eggs d) Beans with ham added

c) Milk and eggs While all the options will provide more protein to the diet, milk, eggs, meat, fish, and poultry provide essential amino acid

Digestion of starch begins in which of the following structures? a) Stomach b) Large intestine c) Mouth d) Small intestine

c) Mouth Digestion of starch begins in the mouth by the action of amylase.

What is the most common mechanism of hormone control? a) Positive feedback b) Hypothalamic-pituitary-adrenal axis c) Negative feedback d) Hypothalamic-pituitary-target cell feedback

c) Negative feedback With negative feedback, the most common mechanism of hormone control, some feature of hormone action directly or indirectly inhibits further hormone secretion so that the hormone level returns to an ideal level or set point.

A client with diabetes mellitus arrives at the hospital with a blood glucose level of 639 mg/dl. What assessment data would indicate type 2 diabetes mellitus rather than type 1? a) Weight loss b) Kussmaul's respirations c) Negative ketones in urine d) Metabolic acidosis

c) Negative ketones in urine Hyperosmolar hyperglycemic state (HHS) occurs most commonly in clients with type 2 diabetes mellitus that is out of control. The elevated glucose level leads to dehydration. But because the client is able to make some insulin, there is no ketoacidosis or compensatory hyperventilation. Weight loss occurs with both diabetic ketoacidosis and HHS because of dehydration.

A client with severe hypoglycemia is unconscious. Which method of providing glucose should be avoided? a) Glucose gel in the buccal pocket b) IM glucagon injection c) Orange juice orally d) Dextrose IV

c) Orange juice orally When clients are unconscious it is not safe to attempt to have them swallow liquids. Alternate routes that reduce the risk of choking such as buccal absorption, intramuscular or intravenous injections are preferred.

A nursing instructor is teaching a group of students about the action of hormones. The instructor determined that teaching was effective when the students' recognize the local action of hormones as: a) Autocrine b) Pancreatic c) Paracrine d) Hormonal

c) Paracrine When hormones act locally on cells other than those that produced the hormone, the action is called paracrine. Hormones also can exert an autocrine action on the cells in which they were produced. Pancreatic and hormonal are not actions.

What body part is the largest serous membrane in the body, constitutes the outer wall of the intestine, and contains a serous fluid between its two layers? a) Omentum b) Haustration c) Peritoneum d) Mesentery

c) Peritoneum The peritoneum is the largest serous membrane, and constitutes the outer wall of the intestine, continuous with the mesentery. The greater omentum helps to prevent infection from entering the peritoneal cavity, and protects the intestines from cold. Haustration is the segmental mixing movements of the large intestine (colon).

A client has developed a ruptured appendix. The nurse is aware the client is at high risk for: a) Diarrhea b) Gastritis c) Peritonitis d) Vomiting

c) Peritonitis Complications of a ruptured appendix include peritonitis, localized peril abscess formation, and septicemia. Vomiting may occur with peritonitis; however, the physiologic effect of peritonitis places the client at highest risk. Diarrhea and gastritis may occur from viral or bacterial infections, food intolerance, or gastric irritation

Which of the following manifestations would a nurse expect when assessing a child with insufficient growth hormone (GH) secretion? a) Lower blood calcium levels b) Increased susceptibility to infection c) Rank below 10% on the growth chart d) Mental sluggishness

c) Rank below 10% on the growth chart GH stimulates growth of bone and muscle and promotes protein synthesis and fat metabolism and decreased carbohydrate metabolism. A child lacking sufficient GH would demonstrate lack of growth but not the mental delays seen in childhood hypothyroidism.

A nurse is completing an abdominal assessment on a patient suspected to have appendicitis. When the nurse applies and then releases pressure in the patient's right lower quadrant, the patient experiences tenderness. The nurse is documenting the presence of which of the following? a) Referred tenderness b) Periumbilical tenderness c) Rebound tenderness d) Peforated appendix

c) Rebound tenderness The nurse documents the presence of rebound tenderness, defined as tenderness that occurs when the nurse applies and then releases pressure to an area.

Fat that is not absorbed in the intestine is excreted in the stool. Which of the following terms is used to describe fatty stools? a) Clay-colored stools b) Diarrhea c) Steatorrhea d) Chyme

c) Steatorrhea Fat that is not absorbed in the intestine is excreted in the stool. Steatorrhea is the term used to describe fatty stools. Chyme is produced in the stomach as proteins are broken down. Diarrhea and clay-colored stools are not caused by poor fat absorption.

A child is born with dwarfism to normal-sized parents. The physician is explaining how growth hormone (GH) plays a central role in the increase in stature that characterizes childhood and adolescence. What is the first step in the growth hormone chain of events? a) The liver is stimulated. b) Epiphyseal growth plates of long bones are influenced. c) The hypothalamus secretes GHRH. d) GH is released and circulates unbound in the plasma.

c) The hypothalamus secretes GHRH. Like other pituitary functions, hypothalamic stimulation precedes hormone release. In the case of GH, stimulation is the result of GHRH by the hypothalamus. GH is then released by the pituitary gland, stimulating the liver to release IGFs, which ultimately causes the epiphyseal plates of long bones to grow.

The nurse explains to a client in labor who has demonstrated ineffective contractions impeding progression of labor that the health care provider has added oxytocin infusion to the orders. Which of these does the nurse teach the client is the purpose of oxytocin? a) To reduce pain of uterine contractions b) To contribute to maturation of fetal lungs c) To stimulate contraction of the uterus d) To protect the fetus from effects of forceful contractions

c) To stimulate contraction of the uterus The role of oxytocin is to stimulate contraction of the pregnant uterus and milk ejection from breasts after childbirth. An infusion of oxytocin will promote effective contractions.

A nurse is instructing a patient about proper nutrition. When the MyPlate model is used for healthy eating, which of the following food groups should the nurse encourage the patient to eat the most of? a) Fruits b) Protein c) Vegetables d) Dairy

c) Vegetables According to Figure 47.6, the MyPlate model demonstrates that the largest portion of food should come from the vegetable food group. The smallest amount of food should come from the dairy and fruit groups. Protein and grains are food groups that are equal in amounts second to vegetables.

Hyperthyroidism that is inadequately treated can cause a life-threatening condition known as a thyroid storm. What are the manifestations of a thyroid storm? (Select all that apply.) a) Very low fever b) Bradycardia c) Very high fever d) Delirium e) Tachycardia

c) Very high fever d) Delirium e) Tachycardia Thyroid storm is manifested by a very high fever, extreme cardiovascular effects (i.e., tachycardia, congestive failure, and angina), and severe CNS effects (i.e., agitation, restlessness, and delirium). The mortality rate is high. Very low fever and bradycardia are not manifestations of a thyroid storm

The poison control nurse is helping calm a hysterical mother who called because her child ingested 10 tablets of B-complex vitamins. The mother keeps repeating over and over again, "Is my child is going to die?" Which of these should the nurse convey to the mother? a) Take the child to the emergency department as these fat-soluble vitamins easily reach toxic levels quickly. b) This type of vitamins contains vitamin K; excess of vitamin K may cause blood clots. c) Water-soluble vitamins are excreted into the urine, making toxicity less likely. d) The child may retain fluid or develop edema for a few days after ingestion.

c) Water-soluble vitamins are excreted into the urine, making toxicity less likely. Fat-soluble vitamins are stored in the body and may reach toxic levels if ingested in amounts greater than what is required by the body. Because the water-soluble vitamins are excreted in the urine, they are less likely to accumulate to toxic levels.

A nurse providing dietary guidance to a patient with celiac disease tells the patient that which of the following foods should be avoided? a) Watermelon b) Homogenized milk c) Wheat bread d) Pork products

c) Wheat bread Patients with celiac disease should avoid ingesting gluten, which is the primary protein in wheat, barley, and rye. The nurse should tell the patient to avoid wheat bread. Watermelon, homogenized milk, and pork products do not contain gluten and therefore do not need to be avoided by a patient with celiac disease.

A client is diagnosed with Addison's disease. What statement by the client indicates an understanding of the discharge instructions by the nurse? a) "Once the symptoms go away, I will be able to stop taking my medication." b) "I should be able to control my condition with diet and exercise." c) "If I have surgery, it will cure me." d) "I will have to take my medication for the rest of my life."

d) "I will have to take my medication for the rest of my life." Addison's disease, like type I diabetes, is a chronic metabolic disorder that requires lifetime hormone replacement therapy. The daily regulation of the chronic phase of Addison disease is usually accomplished with oral replacement therapy, with higher doses being given during periods of stress.

A client developed malnutrition while hospitalized for an illness. The client does not understand how that could have happened. The best response would be: a) "Do not worry; weight loss will not affect you." b) "It is the hospital's fault for not feeding you." c) "Did you tell the nurse you did not like the food?" d) "Malnutrition is also common during illness, recovery from trauma, and hospitalization."

d) "Malnutrition is also common during illness, recovery from trauma, and hospitalization." Protein-energy malnutrition most often occurs secondary to trauma or illness. Kwashiorkor-like secondary protein-energy malnutrition occurs most commonly in association with hypermetabolic acute illnesses, such as trauma, burns, and sepsis

Select the category of hormones that include norepinephrine and epinephrine. a) Glycoproteins b) Steroids c) Peptides d) Amines and amino acids

d) Amines and amino acids Hormones can be divided into three categories: (1) amines and amino acids; (2) peptides, polypeptides, proteins, and glycoproteins; and (3) steroids. The amines include norepinephrine and epinephrine. The second category, the peptides, includes polypeptides, proteins, and glycoproteins. The third category consists of the steroid hormones, which are derivatives of cholesterol.

A client has developed a tumor of the posterior pituitary gland. The client is at risk for problems with secretions of: a) Growth hormone-releasing hormone (GHRH) and dopamine b) Adrenocorticotropic hormone (ACTH) and vasopressin c) Somatostatin and prolactin d) Antidiuretic hormone (ADH) and oxytocin

d) Antidiuretic hormone (ADH) and oxytocin The posterior pituitary secretes ADH and oxytocin/vasopressin, while the anterior pituitary secretes the hormones listed in the other choices.

An elderly patient presents with loose mucousy stools. The nurse suspects the patient has Clostridium difficile. What is a priority assessment for the nurse? a) Ask the patient about his or her normal bowel pattern. b) Ask the patient about his or her fluid intake. c) Ask the patient about the foods he or she has consumed. d) Ask the patient about his or her antibiotic use.

d) Ask the patient about his or her antibiotic use. The diagnosis of C. difficile-associated diarrhea requires a careful history, with particular emphasis on antibiotic use.

A patient has experienced a stroke affecting the reticular formation of the medulla and lower pons. The nurse tells the patient's wife that care must be taken with eating to prevent which of the following? a) Duodenal ulcers b) Steatorrhea c) Gastric atony d) Aspiration pneumonia

d) Aspiration pneumonia Diseases (strokes) that disrupt the brain centers of the reticular formation of the medulla and lower pons disrupt the coordination of swallowing and predispose a person to food and fluid lodging in the trachea and bronchi, which leads to asphyxiation or aspiration pneumonia. The other options are not caused by damage to the brain.

One of the accepted methods of screening for colorectal cancer is testing for occult blood in the stool. Because it is possible to get a false-positive result on these tests, the nurse would instruct the client to do what? a) Eat lots of red meat for 3 or 4 days before the test is done. b) Take 1000 mg of vitamin C in supplement form for 1 week prior to testing. c) Eat citrus fruits at least five times a day for 2 days prior to testing. d) Avoid nonsteroidal anti-inflammatory drugs for 1 week prior to testing.

d) Avoid nonsteroidal anti-inflammatory drugs for 1 week prior to testing. To reduce the likelihood of false-positive tests, persons are instructed to avoid nonsteroidal anti-inflammatory drugs such as ibuprofen and aspirin for 7 days prior to testing, to avoid vitamin C in excess of 250 mg from either supplements or citrus fruits for 3 days before testing, and to avoid red meats for 3 days before testing. The other answers are incorrect.

The nurse and nursing student are caring for a client undergoing a severe stressor with release of epinephrine into the bloodstream. Which of these effects on blood glucose levels does the nurse teach the student epinephrine will cause? a) Gluconeogenesis will occur. b) An unusable form of glucose will be released. c) Hypoglycemia will occur. d) Blood glucose will elevate.

d) Blood glucose will elevate. Epinephrine, a catecholamine, helps to maintain blood glucose levels during periods of stress. Epinephrine causes glycogenolysis in the liver, thus causing large quantities of glucose to be released into the blood.

An 18-year-old client is seen in the emergency department after fainting at school. The client reports severe indigestion, and the nurse notes tooth erosion and an irregular heartbeat. The nurse suspects the client may have developed: a) Easting disorder not otherwise specified b) Anorexia nervosa c) Binge-eating disorder d) Bulimia nervosa

d) Bulimia nervosa The client is exhibiting the complications of bulimia nervosa that result from overeating, self-induced vomiting, and cathartic and diuretic abuse. Dental abnormalities due to the high acid content of the vomitus cause tooth enamel to dissolve. Electrolyte disorders can lead to cardiac arrhythmias. Esophagitis, dysphagia, and esophageal stricture are common due to frequent vomiting.

A patient with a body mass index of 32 would be classified as which of the following? a) Extremely obese b) Underweight c) Statistically healthy d) Class I obesity

d) Class I obesity Class I obesity would be diagnosed based on BMI greater than 30. A normal BMI would be between 18.5 to 24.9

Which of these reflects a positive outcome to interventions provided for a 20-year-old woman undergoing treatment for anorexia nervosa? a) Client requests to weigh herself after every meal. b) Weight is 80% of minimally expected body weight. c) Client refuses to look in the mirror because she has gained weight. d) Client has monthly menses.

d) Client has monthly menses. The individual with anorexia nervosa is expected to maintain a minimally normal body weight (e.g., at least 85% of minimal expected weight). The individual has an excessive concern over gaining weight and how the body is perceived in terms of size and shape, and amenorrhea (in girls and women after menarche). A positive outcome is reflected in less obsession over weight or appearance, and return of menses.

Which of the following criteria about insulin would prompt a diagnosis of type 1 diabetes? a) Insulin not efficiently used b) Large amounts of insulin secreted c) Small amounts of insulin secreted d) Complete failure of insulin secretion

d) Complete failure of insulin secretion In type 1 diabetes there is an absolute lack of insulin due to complete failure of the pancreas. In type 2 diabetes some insulin is produced but may not be properly used.

A client is diagnosed with adrenocorticotropic hormone deficiency (ACTH) and is to begin replacement therapy. Regarding which type of replacement will the nurse educate the client? a) Growth hormone replacement therapy b) Replacement therapy with synthetic thyroid hormone c) Replacement therapy with prolactin d) Cortisol replacement therapy.

d) Cortisol replacement therapy. Cortisol replacement is started when ACTH deficiency is present; thyroid replacement when TSH deficiency is detected; and sex hormone replacement when LH and FSH are deficient. GH replacement is indicated for pediatric GH deficiency, and is increasingly being used to treat GH deficiency in adults.

A patient tells the nurse that he is having diarrhea. Which of the following would the nurse recommend to decrease the diarrhea? a) Diet sodas b) Water only c) Drinks with caffeine d) Electrolyte drink containing sodium and glucose

d) Electrolyte drink containing sodium and glucose Water absorption from the intestines is linked to osmotically active particles, such as glucose and sodium. It follows that an important consideration in facilitating the transport of water across the intestine (and decreasing diarrhea) after temporary disruption in bowel function is to include sodium and glucose in the fluids that are consumed.

Pharmacologic treatment for peptic ulcers has changed over the past several decades. The nurse knows that the goal for pharmacologic treatment is focused on which of the following? a) Neutralizing blood count b) Increasing acid production c) Promoting special diet d) Eradicating Helicobacter pylori (H. pylori)

d) Eradicating Helicobacter pylori (H. pylori) Treatment of peptic ulcer is aimed at eradicating the cause and promoting a permanent cure for the disease. Pharmacologic treatment focuses on eradicating H. pylori, relieving ulcer symptoms, and healing the ulcer crater. Acid-neutralizing, acid-inhibiting drugs and mucosa-protective agents are used to relieve symptoms and promote healing of the ulcer crater. There is no evidence that special diets are beneficial in treating peptic ulcer.

Which of the following pathophysiologic phenomena may result in a diagnosis of Cushing disease? a) Autoimmune destruction of the adrenal cortex b) Malfunction of the HPA system c) Hypopituitarism d) Excess ACTH production by a pituitary tumor

d) Excess ACTH production by a pituitary tumor Three important forms of Cushing syndrome result from excess glucocorticoid production by the body. One is a pituitary form, which results from excessive production of ACTH by a tumor of the pituitary gland. Hypopituitarism and destruction of the adrenal cortex are associated with Addison disease. Disruption of the HPA system is not implicated in the etiology of Cushing disease.

A nurse is teaching a client diagnosed with Crohn disease about potential complications. The most appropriate information for the nurse to include would be: a) Chronic constipation b) Excessive weight gain c) Difficulty swallowing d) Fistula formation

d) Fistula formation Complications of Crohn disease include fistula formation, abdominal abscess formation, and intestinal obstruction. Clients with Crohn disease are at risk for weight loss and/or diarrhea. The disease does not cause difficulty swallowing as it typically is in the bowel.

The stomach secretes two important hormones in the GI tract. One is gastrin. What is the second hormone secreted by the stomach? a) Incretin b) Secretin c) Cholecystokinin d) Ghrelin

d) Ghrelin Ghrelin is a newly discovered peptide hormone produced by endocrine cells in the mucosal layer of the fundus of the stomach. It displays potent growth hormone-releasing activity and has a stimulatory effect on food intake and digestive function, while reducing energy expenditure. The isolation of this hormone has led to new insights into the gut-brain regulation of growth hormone secretion and energy balance. The other hormones are secreted elsewhere in the GI tract.

The nurse is caring for a client who received regular insulin at 7 am. Four hours later the nurse finds the client diaphoretic, cool, and clammy. Which of these interventions is the priority? a) Place the client in the supine position. b) Repeat the dose of insulin. c) Bathe the client with tepid water. d) Give the client a concentrated carbohydrate.

d) Give the client a concentrated carbohydrate. The client is displaying symptoms of hypoglycemia, which include headache, difficulty in problem solving, altered behavior, coma, and seizures. Hunger may occur. Activation of the sympathetic nervous system may cause anxiety, tachycardia, sweating, and cool and clammy skin.

A nurse assessing an 8-year-old child notes that the child is 6 feet tall. Which of the following diagnostic tests will be performed to evaluate the cause of the condition? a) Thyrotropin-releasing hormone stimulation b) Urine for catecholamines c) Antithyroid peroxidase d) Glucose load

d) Glucose load Endocrine testing may include urine collection, blood levels, or tests of stimulation or suppression. A child exhibiting signs of gigantism requires the glucose load test. If the pituitary responds correctly, the GH level will drop. If there is a secreting tumor, the GH level will increase.

A client has been diagnosed with cholecystitis (gallbladder inflammation) that has impaired the normal release of bile. Which of the following gastrointestinal consequences is this client likely to experience? a) Impaired glucose metabolism b) Incomplete digestion of starches c) Inadequate gastric acid production d) Impaired digestion of fats

d) Impaired digestion of fats Bile performs a central role in fat metabolism. Gallbladder disease and the accompanying disruption of normal bile release do not result in impaired digestion of carbohydrates, impaired glucose metabolism, or inadequate gastric acid synthesis.

A client comes to the clinic with fatigue and muscle weakness. The client also states she has been having diarrhea. The nurse observes the skin of the client has a bronze tone and when asked, the client says she has not had any sun exposure. The mucous membranes of the gums are bluish-black. When reviewing laboratory results from this client, what does the nurse anticipate seeing? a) Positive C-reactive protein b) Elevated WBC count c) Increase in sedimentation rate d) Increased levels of ACTH

d) Increased levels of ACTH Hyperpigmentation results from elevated levels of ACTH. The skin looks bronzed or suntanned in exposed and unexposed areas, and the normal creases and pressure points tend to become especially dark. The gums and oral mucous membranes may become bluish-black. The amino acid sequence of ACTH is strikingly similar to that of melanocyte stimulating hormone; hyperpigmentation occurs in greater than 90 percent of persons with Addison's disease and is helpful in distinguishing the primary and secondary forms of adrenal insufficiency.

A 60-year-old male client has presented to his primary care provider to follow up with his ongoing treatment for peptic ulcer disease. What is the most likely goal of this client's pharmacologic treatment? a) Increasing the rate of gastric emptying b) Promoting hypertrophy of the gastric mucosa c) Increasing muscle tone of the cardiac sphincter d) Inhibiting gastric acid production

d) Inhibiting gastric acid production Current therapies for peptic ulcer disease are aimed at neutralization of gastric acid, inhibition of gastric acid (H2 antagonists and proton pump inhibitors), and promotion of mucosal protection. Growth of the mucosa itself, strengthening the gastrointestinal (GI) sphincters, and changing the rate of stomach emptying are not goals of the usual pharmacologic treatments for peptic ulcers.

The diagnosis of type 1 diabetes would be confirmed by: a) Insulin is produced but unavailable for use in the body. b) Insulin is present in large amounts for use by the body. c) Small amounts of insulin are produced daily. d) Insulin is not available for use by the body.

d) Insulin is not available for use by the body. Type 1 diabetes is a catabolic disorder characterized by an absolute lack of insulin. In type 2 diabetes, some insulin is produced.

In addition to facilitating bowel movements, a diet that is high in fiber confers which of the following benefits? a) Removing toxins and metabolic by-products b) Increasing intestinal absorption of vitamins and minerals c) Lowering blood pressure and resting heart rate d) Lowering cholesterol and blood glucose

d) Lowering cholesterol and blood glucose Fiber binds with cholesterol and prevents it from being absorbed by the body as well as lowers blood glucose. It does not directly detoxify the body or increase intestinal absorption of vitamins and minerals. Fiber alone does not cause a reduction in blood pressure or heart rate.

The nurse is caring for a client with chronic renal failure who must begin restricting potassium intake. Which of these foods does the nurse emphasize should be avoided? a) White flour b) Butter c) Seafood d) Potatoes

d) Potatoes Foods containing potassium include meats, poultry, fish, milk and cheese, cereals, legumes, and nuts.

While reviewing the concept of nuclear receptors with a group of pathophysiology students, the instructor uses the example of clients with type 2 diabetes mellitus taking pioglitazone, a thiazolidinedione medication. Because of the peroxisome proliferator-activated receptors (PPARs), the drug has which effect on the clients' diabetes? Select the best answer. a) Increases the clients' metabolic rate, thereby giving them more energy, which results in weight loss. b) Help the body burn fats more effectively thereby preventing any cardiovascular problems like myocardial infarction. c) This medicines increases insulin resistance in muscle and fat and increases the amount of glucose produced by the liver. d) Promote glucose uptake and increase the synthesis of certain proteins involved in fat metabolism, which reduces levels of certain types of lipids.

d) Promote glucose uptake and increase the synthesis of certain proteins involved in fat metabolism, which reduces levels of certain types of lipids. The peroxisome proliferator-activated receptors (PPARs) bind intracellular lipid metabolites and regulate the transcription of genes involved in lipid metabolism and adipose tissue metabolism. Pioglitazone is not given solely to increase metabolic rate or to just burn fats, even though this may occur. Thiazolidimedione medications lower insulin resistance in muscle and fat. They also reduce glucose produced by the liver.

A hospital client with a diagnosis of type 1 diabetes has been administered a scheduled dose of regular insulin. Which of the following effects will result from the action of insulin? a) Initiation of glycogenolysis b) Promotion of fat breakdown c) Promotion of gluconeogenesis d) Promotion of glucose uptake by target cells

d) Promotion of glucose uptake by target cells The actions of insulin are threefold: (1) it promotes glucose uptake by target cells and provides for glucose storage as glycogen; (2) it prevents fat and glycogen breakdown; and (3) it inhibits gluconeogenesis and increases protein synthesis. Glucagon, not insulin, promotes glycogenolysis.

Which of the following is the primary purpose of the mucosal barrier in the gastrointestinal tract? a) Muscle stimulation b) Lubrication of stool c) Food digestion d) Protection from acid secretion

d) Protection from acid secretion The primary purpose of gastric mucosa is resistance to the highly acidic secretion that it produces.

Celiac disease commonly presents in infancy as failure to thrive. It is an inappropriate T-cell-mediated immune response, and there is no cure for it. What is the treatment of choice for celiac disease? a) Removal of sugar from the diet b) Removal of fat from the diet c) Removal of protein from the diet d) Removal of gluten from the diet

d) Removal of gluten from the diet The primary treatment of celiac disease consists of removal of gluten and related proteins from the diet. No other answer is correct.

A client with severe hypothyroidism is presently experiencing hypothermia. What nursing intervention is a priority in the care of this client? a) Active rewarming of the client to increase body temperature rapidly b) Placing the client on a hyperthermia blanket and using heated saline in order to induce vasodilation c) Keeping the client in a hypothermic state in order to prevent renal failure d) Slow rewarming of the client to prevent vasodilation and vascular collapse

d) Slow rewarming of the client to prevent vasodilation and vascular collapse If hypothermia is present, active rewarming of the body is contraindicated because it may induce vasodilation and vascular collapse. Prevention is preferable to treatment and entails special attention to high risk populations, such as women with a history of Hashimoto thyroiditis.

A nurse explains to her patient that food is moved along the gastrointestinal (GI) tract with intermittent contractions that mix the food and move it along. These movements are found in which of the following organs? a) Ileocecal valve b) Internal anal sphincter c) Upper region of the stomach d) Small intestine

d) Small intestine The movements of the GI tract can be either rhythmic or tonic. Rhythmic movements consist of intermittent contractions that are responsible for mixing and moving food along the digestive tract. They are found in the esophagus, the antrum of the stomach, and the small intestine. Tonic movements are found in the lower esophagus, the upper part of the stomach, the ileocecal valve, and the internal anal sphincter.

The physician suspects a client may be experiencing hypofunction of an endocrine organ. Select the most appropriate test to determine organ function. a) Suppression tests b) Imaging studies c) Genetic testing d) Stimulation tests

d) Stimulation tests Stimulation tests are used when hypofunction of an endocrine organ is suspected. Suppression tests are used when hyperfunction of an endocrine organ is suspected. Genetic testing is used for DNA analysis, and imaging may be used as a follow-up after the diagnosis.

The nurse is teaching a basic nutrition course for the community. Which statement should the nurse include about fat-soluble vitamins? a) The fat-soluble vitamins do not require fat for absorption. b) These vitamins are a direct source of energy. c) Excess amounts of these vitamins are excreted in the urine. d) The fat-soluble vitamins are stored in adipose tissue and may reach toxic levels.

d) The fat-soluble vitamins are stored in adipose tissue and may reach toxic levels. The four fat-soluble vitamins are vitamins A, D, E, and K. They require dietary fat for absorption and transport. The nine water-soluble vitamins are vitamin C and the B vitamins, which are thiamine, riboflavin, niacin, vitamin B6, pantothenic acid, vitamin B12, folate, and biotin. Because excess amounts of the water-soluble vitamins are excreted in the urine, it is less likely that they may become toxic to the body, but the fat-soluble vitamins are stored in adipose tissue and may reach toxic levels.

A client who is referred to the endocrinologist's office for an evaluation of his hormone levels asks what regulates the hormone levels. The best response would be that hormone levels in the body are primarily regulated by: a) Exogenous forms of hormones b) The positive feedback loop c) The hypophysial portal system d) The hypothalamic-pituitary-target cell system

d) The hypothalamic-pituitary-target cell system The levels of many of the hormones are regulated by feedback mechanisms that involve the hypothalamic-pituitary-target cell system. Positive feedback control refers to rising levels of a hormone that causes another gland to release a hormone that is stimulating to the first. The hypophyseal portal system connects the supraoptic and paraventricular nuclei of the hypothalamus with the posterior pituitary gland. Exogenous forms of hormones (given as drug preparations) can influence the normal feedback control of hormone production and release.

The obstetrical nurse is caring for a client who has been treated for gestational diabetes. When teaching the client about the causes of gestational diabetes, the nurse should include which of these risk factors in the teaching? a) First birth occurring during the teenage years b) First pregnancy c) Woman who has had a child under 5 pounds d) Woman with a family history of diabetes

d) Woman with a family history of diabetes Gestational diabetes occurs most commonly in African American, Hispanic/Latino American and Native American women. It most frequently affects women with a family history of diabetes, a history of stillbirth or spontaneous abortion, women who previously gave birth newborn with fetal anomaly or had a previous large- or heavy-for-date infant, those who are obese, those of advanced maternal age, or those who have had five or more pregnancies.

During a lecture about the function of the intestine related to food digestion, the faculty mentions that when the students consume foods high in acid, the intestines will: a) stimulate the release of glucagon-like peptide 1 to lower blood glucose levels. b) inhibit the release of glucose-dependent insulinotropic peptide to slow gastric emptying. c) stimulate pancreatic enzymes to release more insulin. d) stimulate the release of secretin which then inhibits release of gastrin.

d) stimulate the release of secretin which then inhibits release of gastrin. Secretin, which is secreted by S cells in the mucosa of the duodenum and jejunum, inhibits gastric acid secretion. The entry of an acid chyme into the intestine stimulates the release of secretin, which inhibits the release of gastrin. Several gut-derived hormones have been identified as having what is termed an incretin effect, meaning that they increase insulin release after an oral glucose load. This suggests that gut-derived factors can stimulate insulin secretion after a high-carbohydrate meal. The two hormones that account for about 90% of the incretin effect are glucagon-like peptide 1 (GLP-1)and glucose-dependent insulinotropic peptide (GIP). Because increased levels of GLP-1 and GIP can lower blood glucose levels by augmenting insulin release in a glucose-dependent manner (i.e., at low blood glucose levels no further insulin is secreted, minimizing the risk of hypoglycemia).


Ensembles d'études connexes

population and growth regulation quiz

View Set

Nursing Fundamentals A PrepU questions

View Set

3328 Data Analytics and Visualization

View Set